Anda di halaman 1dari 54

COMLEX Level 3

Practice Items

Copyright O 2000 National Board of Osteopathic Medical Examiners, Inc.


Level 3 Practice Items

Section I
One-Best-Answer Multiple-choice Items
Directions for Items 1 through 82

For each item in this section, select the ONE best answer from the options available.

A 50-year-old male with a history of a,-antitrypsin deficiency presents to the office complaining of the sudden
onset of right-sided pleuritic chest pain and shortness of breath. Three days prior to presentation he had a
laparoscopic cholecystectomy for cholecystitis. He denies nausea, vomiting and diaphoresis. ECG
demonstrates sinus tachycardia. His vital signs reveal:

Temperature:
Pulse:
Respirations:

The next most appropriate diagnostic study is


(A) ventilation-perfusion scan
(B) radiograph of the chest
(C) CT scan of the chest
(D) MRI of the chest
(E) thoracentesis

2. A 67-year-old female with no significant past medical history presents to the emergency department
complaining of vertigo and double vision for approximately one hour. On presentation she is without
symptoms and has a normal neurological examination. An emergent CT scan of the brain is normal. The
most appropriate management is
(A) alteplase
(B) aspirin
(C) dexamethasone
(D) meclizine
(E) heparin

3. A 20-year-old migrant farmer presents to the emergency department after having punctured his foot on a rusty
pitchfork while cieaning out a pig barn. The patient states that he received his first tetanus toxoid vaccine last
month. Which of the following regimens is most appropriate?
(A) a single dose of tetanus toxoid
(B) two doses of tetanus toxoid
(C) two doses of tetanus toxoid and a single dose of tetanus immunoglobulin
(D) three doses of tetanus toxoid and a single dose of tetanus immunoglobulin
(E) three doses of tetanus toxoid and three doses of tetanus immunoglobulin
Page 2 Level 3 Practice Items

4. You are involved in the care of a 48-year-old female who recently underwent a radical hysterectomy and
bilateral salpingo-oophorectomy for stage 1B cervical cancer. She is very concerned about the risk of
recurrence and questions you regarding what symptoms she should look for. You advise her to watch for
(A) excess thirst and urination
(B) persistent vaginal pain
(C) persistent headaches or visual disturbances
(D) edema of one or both lower extremities
(E) persistent diarrhea

5. A 20-year-old female presents to the emergency department complaining of abdominal pain. She states that
the pain awoke her from sleep 12 hours ago and has been progressively getting worse. The pain was initially
vague and diffuse but has now settled in her right side. She is sexually active and is in the middle of her
menstrual cycle. She has been febrile to 38.3'C (101'F). Physical examination is remarkable for right lower
quadrant abdominal tenderness with guarding and right adnexal tenderness on pelvic examination. Laboratory
studies reveal:

White blood cell count: 13,000/mm3 (13 x 109/L)


Hematocrit: 0.45 (45%)
Differential: normal
HCG: negative

Appropriate management includes


(A) air contrast barium enema
(B) antibiotic therapy and serial abdominal examinations
(C) diagnostic laparoscopy
(D) MRI of the abdomen
(E) pelvic exam under anesthesia with dilatation and curettage

6. A 36-year-old female presents to the office with a two-year history of inflammatory bowel disease 2nd :?.).
back pain. Which of the following would be a contraindication to performing an abdominal I!.rnpha:ic p~rn:?
(A) asthma
(B) cholecystectomy one week ago
(C) congestive heart failure
(D) inflammatory bowel disease
(E) hiatal hernia

A 24-year-old female who is sexually active presents to the office complaining of pain and burning with
urination for one day. She reports no fever or chills and has no allergies to medication. Urinalysis shows too
numerous to count white blood cells. The most appropriate management is
(A) ciprofloxacin
(B) amoxicillin
(C) trimethoprim-sulfamethoxazole
(D) phenazopyridine
(E) erythromycin
Level 3 Practice Items Page 3

8. A 6-month-old female with a temperature of 39'C (102.2") and irritability presents to the emergency
department. Cerebrospinal fluid analysis reveals a white blood cell count of 2500/mm3 (2.5 x 109/L) with
86% neutrophils, protein of 100 mg/dL (1.0 &), and glucose of 45 mg/dL (2.48 mmol/L). The most
appropriate antibiotic is
(A) ceftriaxone
(B) erythromycin
(C) penicillin G
(D) ciprofloxacin
CE) gentamicin

9. A 27-year-old male presents to the office with right-side otalgia, headaches, a unilateral joint click at the
temporomandibular joint (TMJ), and preauricular crepitance on the right side. To confirm a suspected TMJ
dysfunction, the most appropriate test is
(A) MRJ
(B) ultrasound
(C) plain radiography
(D) CT scan
(E) analysis of TMJ synovial fluid

10. As you are attempting to obtain a history from a 20-year-old victim of sexual assault, you notice that the
patient's answers are vague and incomplete. Your best management of this situation is to
(A) forget the details she is not able to provide
(B) encourage her to take her time, and relate her story beginning where she chooses
(C) question her regarding the risky behaviors which resulted in the assault
(D) perform a drug and alcohol screen
(E) skip the history and go on to the examination and collection of evidence

11. Which of the following procedures is most appropriate for a 55-year-old male with three family members who
have a history of malignant colon polyps, one of whom was diagnosed before 50 years of age?
(A) stool samples for blood
(B) sigmoidoscopy
(C) barium enema
(D) digital rectal exam
(E) colonoscopy

2. A 23-year-old previously healthy female presents to the emergency department with the complaint of urinary
frequency, dysuria and urgency. She has no fever, chills, nausea or vomiting. She is in a monogamous
relationship. Physical examination is unremarkable except for mild suprapubic tenderness. Vital signs are
normal, and she is afebrile. Which of the following tests is necessary?
(A) blood chemistry profile
(B) complete blood count
(C) intravenous pyelogram
(D) urinalysis
(E) vaginal culture
Page 4 Level 3 Practice Items

13. You are asked to see an 87-year-old female nursing home resident with a moderate-size sacral pressure sore
that is being treated with local wound care. Physical examination reveals that the wound looks good without
redness or tenderness, but there is serous drainage. The drainage is cultured. Gram's stain reveals scant
enterococci and no white blood cells. The most appropriate treatment would be
(A) intravenous antibiotics
(B) wound debridement
(C) antibiotic-impregnated beads placed in the wound
(D) continued pressure relief
(E) oral antibiotics

14. A 33-year-old computer programmer presents to the office complaining of intermittent numbness and tingling
involving the thumb, index, and middle fingers, and the radial side of the fourth digit of the right hand. She
denies specific trauma. Her symptoms are worsened by prolonged typing. The numbness and tingling
occasionally wake her up at night. The most appropriate diagnostic test is
(A) MRI of the wrist
(B) bone scan
(C) x-rays of the wrist
(D) median nerve biopsy
(E) electrornyogram/nerve conduction tests

15. A 45-year-old male presents with a subacute type of illness with dyspnea, non-productive cough, and low-
grade fever. The duration of symptoms is approximately two to three months. Symptoms have not responded
to oral antibiotics. Physical examination reveals inspiratory crackles on auscultation of the lungs. Laboratory
examination reveals moderate hypoxemia, leukocytosis and elevated erythrocyte sedimentation rate. Chest x-
ray reveals bilateral patchy peripheral infiltrates. Pulmonary function tests reveal reduced lung volumes and
decreased diffusion capacity. Lung biopsy shows bronchiolitis obliterans. The next step in the management
of this patient is
(A) corticosteroids
(B) cytotoxic agents
(C) intravenous antibiotics
(D) observation
(E) rib raising and thoracic pump

16. A 38-year-old female presents to the office stating that she has not had a menstrual period in nine months and
has been having leakage from her breasts. She developed postpartum psychosis after the birth of her child I8
months ago, for which she was placed on thiothixene. The first step in the management of this patient is
(A) blood tests for estradiol and progesterone
(B) CT scan of the brain
(C) discontinuance of thiothixene
(D) low-dose oral contraceptive
(E) thyroid ultrasound
Level 3 Practice Items Page 5

17. A 43-year-old juvenile correctional officer presents to the office after a busy weekend at work. His right leg
is swollen with signs of inflammation and engorgement of the collateral veins in the thigh. Venography is
normal. The most appropriate initial management plan for this patient would include
(A) full anticoagulation therapy
(B) an anti-inflammatory agent
(C) a fibrinolytic agent
@) low molecular weight heparin
(E) effleurage and petrissage of the leg

18. A patient with chronic obstructive pulmonary disease is complaining of chest pain and an inability to take
deep breaths. Which of the following techniques will most likely increase this patient's respiratory motion via
sympathetic stimulation?
(A) cranial treatment of the vault
(B) muscle energy treatment to the OA joint
(C) counterstrain to the AA joint
(D) high velocity low amplitude treatment to T3-T5
(E) lymphatic pump

19. A 23-year-old female presents to the ofice complaining of intermittent bleeding since the initiation of
medroxyprogesterone acetate three months earlier. The most appropriate management is
(A) transvaginal ultrasound
(B) hysteroscopy
(C) dilatation and curettage
(D) another dose of medroxyprogesterone
(E) alternative contraception

20. An 82-year-old male is found to have iron-deficiency anemia. His only significant medical history is a
Billroth I1 procedure for peptic ulcer disease 25 years ago. He has been taking two aspirin daily for
osteoarthritis. Stool is negative for occult blood. The most appropriate initial step in management is to
(A) discontinue aspirin and treat with oral iron
(B) investigate for iron malabsorption
(C) investigate the gastrointestinal tract
(D) treat with antacids and oral iron
(E) treat with intramuscular iron

21. An 18-year-old female, for whom you have recently prescribed oral contraceptives as her first contraceptive
method, calls concerned about the breast swelling which she has noticed during the first week of taking the
pills. Her anxiety is increased because of a family history of breast cancer. You should
(A) encourage the patient to continue with her present pill
(B) change her birth control method
(C) order mammography
(D) add a diuretic
(E) restrict caffeine
Page 6 Level 3 Practice Items

. A previously-well 19-year-old female presents to the emergency department u.ith a complaint of numbness of
the right lower extremity. The numbness occurred suddenly the previous e\.ening after an argument with her
boyfriend. It lasted several hours but has now resolved. Physicai examination is unremarkable. The most
appropriate management is
(A) discharge to her private physician
(B) lumbar puncture
(C) duplex study of the right lower extremity
@) observation in the hospital for 23 hours
(E) MRI of the brain

23. A healthy 80-year-old male asks how often he should have occult blood testing of the stool. He has no family
history of cancer. You advise
(A) twice per year
(B) once per year
(C) once every two years
(D) once every five years
(E) it is not necessary for his age group

24. A 24-year-old female presents complaining that she cannot control her weight. She states that since entering
college six years ago, she has gained 150 pounds. The most important aspect of the patient's history is
(A) family history of cancer
(B) hair growth and smooth skin
(C) dietary intake
(D) rectal bleeding
(E) history of mental illness

25. A 20-year-old femaIe has just had an uncomplicated vaginal delivery following an uneventful first pregnancy.
Screening for IgG rubella antibody at the time of the first prenatal visit was negative. The patient is now
breastfeeding. You should
(A) administer rubella vaccine before the patient leaves the hospitai
(B) delay administration of rubella vaccine until breastfeeding is discontinued
(C) administer human immune gamma globulin
(D) advise the patient to return for rubella vaccine when she is contemplating her next pregnancy
(E) advise the patient to avoid contact with children who have this disease

26. A 16-year-old female presents with her mother for family planning. She is giggling during the history and
uncooperative during the examination. Her mother wants you to advise as to what would be the most
appropriate form of birth control for her daughter. Based on the patient's age, mannerisms, and the mother's
concerns, which of the following would you recommend?
(A) condoms
(B) diaphragm
(C) medroxyprogesterone acetate injection
(D) oral contraceptives
(E) intrauterine device
Level 3 Practice Items Page 7

27. A 59-year-old male who underwent mitral valve replacement with porcine heterograft four months ago
presents t o the office as a new patient. He is very concerned because he has not had warfarin and he was told
by his cousin, who is a nurse, that he could have a stroke. There is no history of stroke or cardiac arrhythmia.
His current ECG is sinus rhythm. You advise him that the therapy he requires is
(A) aspirin, 325 mg daily
(B) warfarin to maintain an INR of 2.0-3.0
(C) observation alone
@) pentoxifylline
(E) low molecular weight heparin

The mother of a 2-month-old infant calls the office for advice. She is concerned that her baby always appears
hungry. The baby weighed eight pounds at birth and presently is eleven pounds. She feeds the baby five to
six ounces of formula every three hours throughout the day. The baby sleeps well at night. The mother has
started rice cereal at the advice of a friend. Appropriate advice to give the mother is to
(A) delay solid foods until six months of age
(B) add fruits and vegetables to the diet
(C) give water when the baby seems hungry
(D) put cereal in the baby's bottle
(E) wake the baby at night for feedings

A 10-week-old female presents to the emergency department with her father who states that the infant is
breathing fast and coughing. Medical history is significant for conjunctivitis shortly after birth, that resolved
with treatment. On physical examination, the patient is afebrile but tachypneic. No wheezing is noted, and
there is no rash or organomegaly. This condition is most likely caused by
(A) respiratory syncytial virus
(B) Streptococcus pneumoniae
( C ) Haemophilus influenzae type B
(D) Chlamydia trachomatis
( E ) Listeria monocytogenes

30. A 22-year-old female medical student presents to the office with a four-year history of increasing dysphagia,
abdominal bloating, and intermittent vomiting. She has dysphagia with solids and liquids. The patient also
reports occasional nocturnal coughing and two episodes of pneumonia in the last year. A barium swallow
reveals irregular mucosa in the distal esophagus. The most appropriate diagnostic test is
(A) upper endoscopy
(B) esophageal manometry
(C) 24-hour esophageal pH monitoring
(D) gastroduodenal manometry
(E) CT scan of the chest
Page 8 Level 3 Practice Items

31. On admission to the nursing home, an alert, cooperative and spry-appearing 82-year-old female undergoes
mental status testing consisting of a Folstein Mini Mental State Examination, in which a score of 24 or less
suggests cognitive impairment, and a Geriatric Depression Scale, in which a score of 14 or greater indicates
depression. The patient scores 22 on the Folstein and 3 on the Geriatric Depression Scale. You would
(A) enter a diagnosis of organic brain syndrome on the chart
(B) enter a diagnosis of major depression on the chart
(C) obtain consultation with a psychiatrist before entering a diagnosis
(D) repeat the Folstein in one month before entering a diagnosis
(E) enter a diagnosis of senile dementia of the Alzheimer's type on the chart

32. A 42-year-old male presents to the office with a family history of glaucoma. He has diabetes mellitus. He
should have an eye examination, including glaucoma screening, every
(A) six months
(B) one year
(C) two years
(D) three years
(E) five years

33. A 35-year-old female presents to the office complaining of hoarseness. On physical examination, there is a 2-
cm nodule in the left lobe of the thyroid gland. There is no cervical lymphadenopathy. She is clinically
euthyroid. The next most appropriate imaging procedure is
(A) radiographs of the neck
(B) CT scan of the neck
(C) MRI of the neck
(D) gallium scan of the neck
(E) radioiodine scan of the neck

34. A 5-year-old male is rushed to the office by his parents who found him moaning, holding his head, and crying
inconsolably. Examination reveals a semi-stuporous child who responds to painful stimuli and does not
recognize his parents or respond to voice commands. Vital signs reveal:

Temperature: 39.7'C (103.4'F)


Pulse: 160/min
Blood pressure: 80/45 mm Hg
Respiratory rate: 30lmin

Eye grounds do not reveal papilledema. No lateralizing signs are present. Positive Kernig's and Brudzinski's
signs are present and nuchal rigidity is noted. The most useful test in the evaluation would be
(A) CT scan of the brain
(B) electroencephalogram
(C) lumbar puncture
(D) blood culture
(E) liver function tests
Level 3 Practice Items Page 9

35. An 8-year-old female is brought to the office by her parents. They report that she was recently labeled by her
teachers as "dyslexic." The parents ask for your recommendations for treatment. Physical examination is
normal. Her vision is 20120. The most appropriate management is
(A) recommendation to be placed in special education classes
(B) referral for educational and learning disabilities testing
(C) referral for play therapy
(D) a stimulant medication
(E) to tell the parents that this is a normal developmental step

36. A 32-year-old male presents to the office with complaints of fever, weight loss, chronic diarrhea and
abdominal pain. Physical examination reveals iritis, erythema nodosum, and muscle tension of the quadratus
lumborum bilaterally. The most likely diagnosis is
(A) inflammatory bowel disease
(B) duodenal ulcer
(C) Zenker's diverticulum
(D) HIV gastroenteropathy
(E) non-ulcer dyspepsia

37. A 15-year-old male presents to the emergency department vomiting and confused. On physical examination
he is noted to be tachycardic with decreased skin tone, and he has a fruity odor on his breath. Appropriate
management of this patient would include rehydration and administration of
(A) insulin
(B) chlorpromazine
(C) lorazepam
(D) propylthiouracil
(E) naloxone

38. A 25-year-old sexually active female presents to the office complaining of burning with urination. Urine
dipstick is negative for leukocyte esterase, nitrites, glucose, bacteria, and squarnous epithelial cells. The next
diagnostic test to perform is
(A) KL'B
(B) renal ultrasound
(C) straight catheter urine sample and repeat urine dipstick
(D) ultrasound of the bladder
(E) cervical culture

39. A sexually active 15-year-old female presents with active vaginal bleeding due to an incomplete abortion. She
requires diIatation and curettage. Although her parents are paying the bill, she refuses to let you tell them
why she is being admitted to the hospital. An appropriate plan of action at this point is to
(A) refuse to be her physician because of the complicated family dynamics
(B) perform a dilatation and curettage even without parental consent
(C) treat her as an outpatient, since the parents can find out why she is admitted by requesting her records
iD) admit her, but postpone dilatation and curettage until a social worker handles the family issues
(E) consult with your attorney in case litigation is brought against you by the parents
Page 10 LeveI 3 Practice Items

40. A 26-year-old patient has just taken a job as a carpet layer and has heard that this job predisposes him to knee
injuries. He would like to know what he can do to prevent any problems. What advice would you give?
(A) wear a chondromalacia brace to avoid wearing down the knee cap
(B) wrap the knee in an elastic bandage to avoid suprapatellar bursae distention
(C) wear anterior knee pads to prevent development of infrapatellar bursitis
@) tape the medial knee daily to avoid development of pes anserinus bursitis
(E) wear elastic stockings under trousers to avoid the development of Baker's cyst

41. An 8-year-old female presents with her mother complaining of spots of biood on the child's underwear. On
examination a tear in the posterior vaginal introitus is noted, but the child will not answer historical questions.
The obligation to this patient is to
(A) immediately refer to a psychiatrist
(B) call the police to come to the office immediately
(C) call a family meeting
(D) call child protective services
(E) entrust the child to the mother's care to investigate the situation

A 45-year-old male alcoholic presents to the emergency department with stomach pain. Laboratory data
reveals: hemoglobin = 10 gldL (6.3 mmol/L) and MCV = I10 p3 (1 10 fL). What is the most likely cause of
this patient's anemia?
(A) iron deficiency
(B) folate deficiency
(C) chronic disease
(D) pyridoxamine deficiency
(E) sickle cell anemia

43. A 22-year-old female presents to the emergency department with a stab wound of the right chest. She is
apneic, has a blood pressure of 80140 mm Hg, and exhibits dullness to percussion on the right hemithorax.
The first step in her management is
(A) volume replacement
(B) central venous line
(C) immediate thoracotomy
(D) insertion of a chest tube
(E) endotracheal intubation

44. A 30-year-old maie presents with a history of left testicular swelling which he noticed about a week ago. He
denies any other symptoms. Physical examination reveals a left testicular mass which is firm in consistency.
No tenderness is noted. The next appropriate initial step in diagnostic testing is
(A) CT scan of the abdomen
(B) measurement of a-fetoprotein
(C) measurement of human chorionic gonadotropin
(D) testicular biopsy
(E) testicular sonogram
Level 3 Practice Items Page 11

45. A 3-lb premature female develops jaundice at 24 hours of age. The infant's bilirubin levels are drawn twice
daily. The levels continue to climb, and phototherapy starts at 48 hours of age. The infant is lethargic,
feeding poorly, and there is a loss of Moro's reflex. The infant is not responding to the phototherapy. An
exchange transfusion is performed. This procedure would prevent
(A) neonatal hepatitis
(B) hyperbilirubinemia
(C) septicemia
(D) major blood group incompatibility
(E) kernicterus

46. A 42-year-old gravida 4 para 4 female in a monogamous relationship presents desiring a convenient, highly
effective, reversible form of contraception unrelated to sexual intercourse. She has a history of severe
migraine headaches and admits to smoking one pack of cigarettes each day. Which of the following forms of
contraception would be most appropriate to recommend to this patient?
(A) condoms
(B) medroxyprogesterone
(C) intrauterine device
(D) oral contraceptives
(E) sterilization

47. A fourth-year medical student accidentally jabbed himself deeply in the palm with a needle after having
performed a venipuncture on a young adult male. The student immediately washed his hand with soap and
water and called the nursing supervisor to report the incident. The source patient has never been tested for
HIV, although he admits to intravenous drug use and bisexual activity in the past. The patient consents to be
tested for HIV and hepatitis B. The student should be offered
(A) HIV testing at 6, 12, and 24 weeks and post-exposure chemoprophylaxis if the need arises
(B) immediate zidovudine therapy
(C) zidovudine therapy pending the result of the source patient's HIV test
(D) zidovudine and 3TC therapy pending the result of the source patient's HIV test
(E) immediate zidovudine and 3TC therapy, plus a protease inhibitor

48. A 49-year-old male presents to the office with complaints of severe headaches that are localized to the right
frontal area. They wake hm up at night and are unrelieved by aspirin. He has a history of lung cancer. A
CT scan of the brain reveals a large right frontal mass with a slight shift to the ventricles and evidence of
cerebral edema. The most appropriate management is
(A) emergency brain biopsy
(B) lumbar puncture
(C) intrathecal chemotherapy
(D) dexamethasone
(E) morphine sulfate
Page 12 Level 3 Practice Items

49. A 30-year-old female presents to the office with a mass in her left breast which she discovered on self-
examination. She is currently menstruating; she has no other complaints. Her family history is negative for
breast cancer. On physical examination, the mass is in the left upper outer quadrant, about 3 cm in size, and
movable. She denies any changes in the mass during menses. Ultrasound of the breast reveals that the mass
is solid. The most appropriate management is
(A) weekly lymphatic pump treatments and re-evaluate in six months
(B) repeat the ultrasound when she is not menstruating
(C) CT scan of the chest
(D) MRI of the chest
(E) surgical consultation

50. A local attorney contacts you to testify on behalf of his client. The child, who has a partially controlled
seizure disorder, had been denied admission to the school archery team. He asks that you testify that children
with partially or poorly controlled seizures have the right to participate in all non-contact sports and some
limited-contact sports. After a review of the literature, you explain that your testimony would be that a
seizure patient may participate in
(A) any non-contact sport
(B) any contact sport providing he wears protective headgear
(C) any sport where a seizure is unlikely to injure himself or others
(D) any sport, by constitutional law
@) no sports, due to personal injury potential

51. A 7-year-old male is brought to the emergency department by his mother. She reports a 12-hour history of
fever, increasing lethargy, and rash. The child is noted to be confused and combative. Vital signs reveal a
temperature of 40.3"C (104.6'F), heart rate of 170lmin, respirations of 45lmin, and blood pressure of 80120
m m Hg. He is noted to have purpura of his extremities and widespread petechiae. The most useful diagnostic
procedure would be
(A) blood cultures
(B) coagulation studies
(C) complete blood count
(D) CT scan of the brain
(E) lumbar puncture

52. A 76-year-old female presents to the office complaining of frequent falls at home. Her medications include
ranitidine, amitriptyline, fluoxetine, verapamil, and ibuprofen. A history of lightheadedness is obtained with
an orthostatic decrease in blood pressure from 150170 mm Hg sitting to 100/50 mm Hg standing. The drug
effect that is most likely responsible for this change in blood pressure is
(A) cholinergic blockade secondary to the amitriptyline
(B) adrenergic blockade secondary to the verapamil
(C) serotoninergic blockade secondary to the fluoxetine
(D) histaminic blockade secondary to the ranitidine
(E) diuresis secondary to the ibuprofen
Level 3 Practice Items Page 13

53. A 56-year-old female presents for evaluation of postmenopausal symptoms of hot flashes and vaginal dryness.
An increased kyphosis in the upper thoracic spine is noted. Proper diagnostic follow-up would include
(A) evaluation of nutritional, hormonal and activity status
(B) imaging studies of the hip joint to evaluate osteoporosis
(C) Iaboratory studies to include lipid panel
(D) sedating medications for adequate rest
(E) bone scan to rule out metastatic disease

54. A 30-year-old hiker asks your advice regarding prevention of Lyme disease. You recommend
(A) vaccination
(B) prophylactic antibiotics
(C) routine serum testing
(D) mosquito repellant
(E) avoidance of swampy areas

55. A 65-year-old male presents to the emergency department after a syncopal episode. It was very sudden with
no warning. There was no bowel or bladder incontinence or residual abnormalities. A neurological
examination is within normal limits. Past medical history is remarkable for depression, for which he is
currently taking amitriptyline. Which of the following abnormalities may be seen?
(A) prolonged PR interval
(B) prolonged QT interval
(C) presence of U waves
(D) peaked T waves
(E) narrowing of the QRS complex

56. A 4-year-old child with Pseudomonas aeruginosa pneumonia is put on gentamicin and piperacillin therapy.
The patient is being sent home on intravenous antibiotics for three weeks. Besides weekly complete blood
counts and creatinine and gentamicin levels, which of the following should be monitored?
(A) creatinine clearance levels
(B) visual field testing
(C) audiogram
(D) liver function studies
(E) piperacillin blood levels

57. A 33-year-old female presents to the office complaining of urinary incontinence associated only with the urge
to void. It is not associated with dysuria or frequency, and has been present for about six weeks. She denies
loss of urine with Valsalva's maneuver. Urinalysis is negative. The most useful diagnostic test would be
(A) post-void residual volume determination
(B) intravenous pyelogram
(C) urethrocystoscopy
(D) urine culture
(E) urodynamic testing
Page 14 Level 3 Practice Items

58. A 13-year-old male presents for a routine health maintenance examination. On physical examination, you note
his height to be at the fifth percentile. There is some testicular enlargement; otherwise, the examination is
unremarkable. The mother is concerned that he will be short. The most appropriate initial step in evaluation
of his short stature should be
(A) testosterone level
(B) assay of human growth hormone
(C) karyotype
(D) thyroid function assessment
(E) x-rays for bone age

59. A 56-year-old female with a history of bladder carcinoma presents with nausea and vomiting, a temperature of
38.9"C(102.0°F), anorexia, and foul-smelling vaginal discharge. She is status post-colostomy and cystectomy
with diverting urostomy and total abdominal hysterectomy. Blood and urine cultures are positive for
vancomycin-resistant Enterococcus. She is admitted to the hospital in an isolation room. Which of the
following is most helpful in controlling the spread of the Enterococcus to other patients?
(A) prophylactic treatment of all patient contacts
(B) use of gloves and hand washing
(C) use of gowns and masks
(D) surveillance cultures of other patients
(E) reducing skin colonization with triple antibiotics

60. A 49-year-old obese male presents to the office complaining of morning headaches, irritability, daytime
sleepiness and the inability to concentrate on his work. His wife reports that he snores loudly. An ECG and
basic chemistry panel are also unremarkable. Physical examination reveals redundant pharyngeal tissue. The
next appropriate step would be a
(A) referral to a sleep lab
(B) nasopharyngoiaryngoscopy
(C) pulse oximetry at night
(D) referral to an allergist
(E) sinus x-ray series

61. A 38-year-old male presents to the office complaining of slowly progressive low back pain. He describes the
pain as constant and associated with nagging stiffness. It is somewhat relieved by walking around. The pain
often radiates into the gluteal area. Osteopathic structural examination reveals marked paravertebral muscle
spasm in the involved area bilaterally, along with multiple tender triggerpoints. The next step in diagnosis is
(A) sacroiliac joint plain x-rays
(B)
(C) myelogram
(D) radionuclide examination
(E) CT scan of the pelvis
Level 3 Practice Items Page 15

62. A 41-year-old painter complains of severe left low back pain after lifting a dresser. He states that the pain
"shoots" down his leg into his toes. On physical examination he is unable to dorsiflex his great toe against
resistance. He has a positive straight leg raising test at 30" Which of the following muscles would be
expected to be weak?
(A) tibialis anterior
(B) flexor digitorurn longus
(C) peroneus brevis
(D) peroneus longus
(E) extensor hallucis longus

63. A 35-year-old female presents to the office with complaints of heavy vaginal bleeding and breakthrough
bleeding for the last five menstrual cycles. After thorough examination and testing, you determine that the
patient is anovulatory because her serum progesterone level is not elevated and she does not have a biphasic
basal body temperature. To determine the cause of her menometrorrhagia the next most useful test to obtain
is
(A) endometrial biopsy
(B) laparoscopy
(C) vaginal ultrasound
(D) CA-125 level
(E) vaginal cultures

64. A 64-year-old male complains of numbness and cramping in both feet with minimal ambulation. This has
been progressive for years. He has a history of myocardial infarction, cerebrovascular accident, and
hypertension. On examination both feet are pale and cool. Distal pulses can only be detected by Doppler
scan. He has an aortic bruit and bilateral femoral bruits. Which of his daily habits has the greatest effect on
the progression of his disease?
(A) high-fat diet
(B) sedentary lifestyle
(C) medication noncompliance
(D) cigarette smoking
(E) ethanol ingestion

65. A 65-year-old male presents to the office noting that he has been feeling "run down," and worried because he
is gaining weight, even though he is eating the same amount. He states that he hasn't been able to take his
daily walk because his legs have felt too weak and have been swelling. He is currently taking lisinopril and
fluvastatin. On physical examination, he is lethargic-appearing. His legs aren't particularly weak, but he does
have a delayed Achilles reflex. The most appropriate laboratory testing includes
(A) BUN and creatinine
(B) lipid panel and ALT
(C) magnesium and calcium
(D) TSH level
(E) vitamin B,,and folate
Page 16 Level 3 Practice Items

66. A 4-year-old female with a known brain stem carcinoma delelops right trigeminal neuralgia. She has tried
codeine and NSAIDs to control the pain without benefit. The next most appropriate management is
(A) morphine sulfate
(B) rhizotomy of the nerve
(C) chemotherapy
(D) carbamazepine
(E) corticosteroid nerve injection

A 44-year-old male has gained weight, lost energy, and developed polyuria. On further questioning, you find
that the patient's father and two uncles have type 2 diabetes mellitus. The patient's fasting plasma glucose is
135 mg/dL (7.4 mmoVL). Which of the following is the best course of action in advising and treating this
patient?
--

(A) 1500-calorie diet


(B) a diet low in carbohydrates and fats
(C) low-dose insulin therapy
(D) a diet high in carbohydrates and low in fats combined with an exercise program
(E) bedtime insulin and an oral agent in the morning

68. A 61-year-old male complains of blurred vision, rainbows around lights, ocular pain, and nausea. One eye is
red with a fixed mid-dilated pupil. The most appropriate management is to
(A) prescribe topical cycloplegics
(B) immediately refer to an ophthalmologist
(C) prescribe topical antibiotic drops
@) prescribe topical steroid drops
(E) recommend warm compresses

69. A 35-year-old female presents to the office complaining of widespread pain and tenderness associated with
fatigue and frequent headaches. You diagnosis her with fibromyalgia. The most appropriate management is
(A) avoidance of exercise
(B) corticosteroids
(C) patient education
(D) high velocity low amplitude treatment
(E) splints for the most affected areas

70. A 6-month-old female presents to the emergency department with multiple healing fractures. The patient was
born with fractures to both clavicles and the right humerus as a result of delivery. Hypermobility of the joints
is noted. The most likely etiology of the fractures is
(A) poor obstetrical care
(B) rickets
(C) osteogenesis irnperfecta
(D) child abuse
(E) a strict vegetarian mother
Level 3 Practice Items Page 17

71. A 35-year-old male presents to the office because he blacked out while sledding in his front drive. The
patient states that he is accident-prone and has frequent injuries. During physical examination, you notice a
black eye on the right, old bruises on his neck and right shoulder, and healed scratches on his left neck and
shoulder, He is vague as to when and how he obtained these injuries. The most appropriate management is
to
(A) bring in his spouse to clarify the history of the wounds
(B) bring in his spouse and ask them both about violence
(C) talk to him about taking his spouse to counseling
(D) provide information and supportive care
(E) refer the family to adult protective services

72. A 45-year-old female presents for her annual physical examination and states that she has had progressive loss
of smell and taste sensation over the past six months. The patient is currently in the recovery phases of
chemotherapy for stage 111 Hodgkin's lymphoma. She has no other complaints. Physical examination is
unremarkable with the exception of the loss of taste and smell sensation and glossal candida. The most
appropriate initial management is
(A) referral to an occupational therapist
(B) treatment of the glossal fungal infection
(C) initiation of zinc and vitamin therapy
(D) initiation of artificial saliva therapy
(E) CT scan of the brain

73. A 32-year-old female presents to the emergency department via ambulance immobilized on a long spine board
with a cervical collar in place. She was crossing a street and was stuck by an automobile. Vital signs reveal
a temperature of 35.9"C (96.7"F), respirations of 6/min, a pulse of 140/min, and a blood pressure of 90160 mm
Hg. The patient makes incomprehensible sounds, does not respond to painful stimuli, and is noted to have a
depressed skull fracture. The most appropriate initial therapy is
(A) administration of intravenous 0.9 normal saline
(B) administration of mannitol
(C) placement in reverse Trendelenburg position
(D) administration of intravenous phenobarbital
(E) endotracheal intubation

A 4-week-old male presents to the emergency department with a low-grade fever, cough, and decreased
appetite. On physical examination the child is tachypneic and using accessory muscles of respiration.
Bilateral expiratory wheezing is noted. Which of the following tests will help guide inpatient therapeutic
options?
(A) urinalysis
(B) erythrocyte sedimentation rate
(C) nasopharyngeal wash
(D) serum electrolytes
(E) white blood cell count
Page 18 Level 3 Practice Items

75. A previously healthy 9-month-old child is brought to the emergency department for evaluation of "fast
breathing." The child has a temperature of 38°C (100.4"F). The respiratory rate is 38lmin. The child has
copious clear rhinorrhea and mild wheezing. Chest x-ray demonstrates hyperinflation without infiltrate. Pulse
oxirne'try on room air is 90%. Treatment, in addition to supplemental oxygen, should include
(A) immediate intravenous antibiotics
03) thoracic pump weekly
(C) nebulized albuterol
(D) nebulized ipratropium bromide
(E) oral antibiotics

76. A 17-year-old female presents to the office accompanied by her mother, who states that the patient has
stopped having menstrual periods. History and physical examination reveal that the patient is greater than
15% below her ideal body weight, she exercises four hours each day, and she thinks that she needs to lose
more weight to look good for graduation. Which of the following sources wouid be helpful?
(A) a psychiatrist
(B) a dietician
(C) a gynecologist
(D) an exercise physiologist
(E) a pastoral counselor

A 2-week-old infant is referred to you for evaluation of an abnormal thyroid screening test. You discover that
the test was performed at 12 hours of age, since the mother insisted on early discharge. The T, level was low
and the thyroid-stimulating hormone level was greater than 100 pU/mL. Examination of the infant is normal.
Initial growth and development have been normal. Appropriate management is to
(A) start thyroxine
(B) perform a thyrotropin-releasing hormone stimulation test
(C) send a T, and thyroid-stimulating hormone test to the hospital laboratory
@) discount the results as erroneous since the infant's growth and development have been normal
(E) obtain a thyroid scan to rule out an ectopic gland or complete absence of the thyroid

78. A 55-year-old male office worker consults you for evaluation of chronic low back pain. Physical examination
is negative except for obesity. Upon questioning, you discover that both he and his wife enjoy snacking on
fast foods and snacks. In addition to nutritional counseling and recommendations regarding exercise, you
suggest which of the following psychological interventions?
(A) hypnotherapy
(B) psychoanalysis
(C) marital therapy
(D) cognitivehehavioral therapy
(E) insight-oriented psychotherapy
Level 3 Practice Items Page 19

79. A 60-year-old female with chronic renal failure presents to the office complaining of generalized fatigue. She
has no other complaints. Physical examination is unremarkable and laboratory results reveal:

Hemoglobin: 8.0 g/dL (4.9 mmolL)


Hernatocrit: 0.24 (24%)
White blood cell count: 5,4001 mm3 (5.4 x 109/L)
Platelet count: 150,000/mm3 (150 x 109/L)
Ferritin level: 170 mg/rnL (170 p g L )
Vitamin B,, and folate levels: normal

The most appropriate management is


(A) parenteral iron
(E3) parenteral vitamin B,,
(C) transfusion with two units of packed red blood cells
(D) parenteral erythropoietin
(E) oral folate

80. A 57-year-old smoker presents with hoarseness and a non-tender 3-cm firm mass in the right side of the neck.
The next appropriate step is
(A) CT scan of the neck
(B) fine needle aspiration biopsy of the mass
(C) laryngoscopy
(D) MRI of the neck
(E) open excisionai biopsy of the neck

81. A 45-year-old obese male begins an exercise program. You tell him that by walking for 30 minutes at a 4
- ~ p k - p a c e h ~ i l ~ h u r n - a p p r a x i m a t & _ 2 0calories.
0 If he is
intake by 200 calories, he can expect to lose a pound of fat in
(A) 3 days
(B) 5 days
(C) 7 days
(D) 10 days
(E) 14 days

82. A 65-year-old male with small cell lung carcinoma presents to the office complaining of weakness and fatigue.
Physical examination is unremarkable. Laboratory values reveal a sodium of 120 rnmol/L. The most
appropriate management would be
(A) corticosteroids
(J3) water restriction
(C) oral salt tablets
(D) IV 0.45% saline
(E) furosemide
Page 20 Levei 3 Practice Items

Section I1
Multiple-Choice Clinical Set Problems
Directions for Items 83 throueh 200

For each item in this section, select the ONE best answer from the options available. Each item is identified
as being part of a larger case.

CASE A

A 30-year-old female presents with a complaint of fatigue. She states that she has been waking up more and
more tired each morning. She has not been going to work or socializing. - She also complains of a sore chest
and tender arms. Physical examination is normal except for diffuse swelling of the extremities with no
apparent swelling of the joints. She has six sets of symmetrical tenderpoints located at the clavicular heads,
mid trapezius muscle, medial elbows, medial scapular border, and at the posterior superior iliac spine.

83. The most likely diagnosis is


(A) Lyme disease
(B) chronic fatigue syndrome
(C) rheumatoid arthritis
(D) polymyositis migrans
(E) fibromyaigia syndrome

84. Which of the following diagnostic tests is likely to be ABNORMAL in this patient?
(A) rheumatoid factor
(B) HLA-B27
(C) sleep study
(D) erythrocyte sedimentation rate
(E) synovial fluid evaluation

85. Which of the following medications would be most useful in treating this patient?
(A) diazepam
(B) cyclobenzaprine
(C) carbidopa
(D) temazepam
(E) haloperidol

86. Which of the following exercise regimens would be most helpful for this patient?
(A) no exercise until fatigue resolves
(B) gentle aerobic exercise
(C) isometric exercise
(D) weight training
(E) high intensity endurance training
Level 3 Practice Items Page 21
- --

CASE B

A 76-year-old female presents to the office with a chief complaint of constipation which she has treated
successfully for 20 years with over-the-counter laxatives. Over the past few months, however, the constipation
has gotten progressively worse. She denies any change in the color of her st001 but states that she feels
bloated and has had intermittent pelvic pain.

87. On rectal examination, you note that the patient has heme-positive stools. The most appropriate next step is
to
(A) refer the patient for a colonoscopy
(B) order a serum carcinoembryonic antigen
(C) perform an anoscopy
(D) tell the patient to stop eating red meat and repeat hemoccult testing
(E) order a pelvic ultrasound

88. Treating the somatic dysfunction in a viscerosomatic reflex would have which of the following effects?
(A) correction of the visceral dysfunction
(B) reduction of somatic symptoms
(C) no effect
(D) restoration of homeostasis
(E) prevention of metastasis

CASE C

A 25-year-old female presents to the office complaining of feeling depressed. She states that this seems to
occur every winter and improves by the spring.

89. This patient is most likely to respond to


(A) insight-oriented therapy
(B) electroconvulsive therapy
(C) pharmacotherapy
(D) light therapy
(E) behavior therapy

90. The most likely etiology of this disorder is


(A) cold weather
(B) influenza virus
(C) increased levels of norepinephrine
(D) decreased daylight hours
(E) sleep deprivation
Page 22 Level 3 Practice Items

CASE D

A 35-year-old male presents to the office with a recent history of nausea, anorexia and vomiting. He also has
noticed resting tremors as well as difficulty with speech and swallowing. Physical examination reveals a large
palpable nodular non-tender liver and chronic tissue texture changes at T6-T8 on the right. He also has
evidence of gynecomastia and testicular atrophy. On eye examination he has golden deposits on the corneal
surface.

91. The most likely diagnosis is


(A) hemochromatosis
(B) viral hepatitis
(C) Wilson's disease
(D) primary biliary cirrhosis
(E) alcoholic cirrhosis

92. The most appropriate diagnostic test is


(A) a-fetoprotein
(B) hepatitis B titer
(C) antimitochondrial antibody level
(D) ceruloplasmin level
(E) ferritin level

93. The most effective therapy would be


(A) liver transplant
(B) interferon
(C) penicillamine
(D) frequent phlebotoay
(E) alcohol consumption restriction

CASE E

A 25-year-old female is brought into the emergency department via ambulance following a motor vehicle
accident. She complains of left shoulder pain. On examination she is alert and oriented but is not taking deep
breaths. Her vital signs reveal:

Respirations: 22lmin
Pulse: 120lmin
Temperature: 37.2-C (99°F)
Blood pressure: 90160 mm Hg

Her heart is rapid but regular in rate. Lung sounds are diminished in the upper and lower fields. She has
ecchymosis and tenderness medial to the shoulder. A chest x-ray and shoulder view is obtained.
Level 3 Practice Items Page 23

94. A common and potentially serious unrecognized injury in this patient could be
(A) aortic arch injury
(B) brachial plexus nerve injury
(C) esophageal injury
(D) thoracic duct injury
(E) tracheobronchial injury

95. Which of the following would be the best tool to identify this injury?
(A) aortogram
(B) barium esophagogram
(C) bronchoscopy
(D) lymphangiogram
(E) MRI of the shoulder

CASE F

A healthy 20-year-old male failed his mid-term examinations. Upon receiving his grades he became hopeless
and decided that he would rather die than show his parents his grades. His friend brings him to the
emergency department because he is threatening to kill himself. He refuses to contract for safety.

96. The most likely diagnosis is


(A) adjustment disorder with anxiety
(B) adjustment disorder with depressed mood
(C) dysthymic disorder
(D) major depressive disorder
(E) schizophrenia

97. The most appropriate initial treatment is


(A) electroconvulsive therapy
(B) outpatient treatment
(C) pharmacotherapy
(D) psychiatric hospitalization
(E) psychotherapy

98. You are concerned that the patient is unable to contract for safety. He is upset because he will not be able to
finish his semester. You should
(A) allow him to go to outpatient treatment
(B) notify his parents
(C) pursue involuntary commitment
(D) prescribe medication
(E) use electroconvulsive therapy
Page 24 Level 3 Practice Items

CASE G

A 40-year-old mildly obese female complains of right upper quadrant abdominal pain and nausea. Palpation
of the abdomen reveals right upper quadrant tenderness. Musculoskeletal evaluation reveals tissue texture
abnormality at T9 on the right.

99. Which of the following diagnostic tests would be appropriate at this time?
(A) ultrasound of the gallbladder
(B) CT scan of the abdomen
(C) upper gastrointestinal x-ray
(D) upper gastrointestinal endoscopy
(E) endoscopic retrograde cholangiopancreatography

100. You wish to treat the T9 area with high velocity osteopathic manipulative treatment. Which of the following
physical findings would suggest that high velocity low amplitude treatment is appropriate?
(A) ability to engage a distinct, solid barrier
(B) tissue texture abnormality
(C) decrease in range of motion at T9
(D) paraspinal muscle hypertonicityltenderness
(E) type I1 motion mechanics

CASE H

A 40-year-old male presents with persistent nausea and vomiting for one month. An upper gastrointestinal
series reveals a gastric ulcer. He is treated with cimetidine without improvement. He also admits to a 30-
pound weight loss over the past month.

101. Musculoskeletal findings in this patient would most likely be palpated at


(A) TI-T3
(B) T5-T7
(C) T9-TI 0
(D) T12-L1
(E) L4-L5

102. The most appropriate step at this time is


(A)addition of antibiotic therapy
(B)repeat upper gastrointestinal series
(C)gastroscopy
(D)CT scan of the abdomen
@) serum gastrin level
Level 3 Practice Items Page 25

CASE I

A 55-year-old male has suffered from heartburn for nearly six months. He treats himself symptomatically
with over-the-counter antacids with fairly good relief, but he recently has had more heartburn symptoms,
infrequent hoarseness, and new onset of difficulty in swallowing meat. He has been using more antacids, and
his symptoms are worse with recumbency.

103. The most likely cause of his symptoms is


(A) Schatzki's ring
(B) Zenker's diverticulum
(C) globus pharyngeus
(D) amyotrophic lateral sclerosis
(E) cricopharyngeal achalasia

104. The most appropriate diagnostic study is


(A) barium swallow
(B) esophageal manometry
(C) esophageal pH monitoring
(D) technetium-99m scan
(E) CT scan of the chest

CASE J

A 65-year-old male is sent to you for progressive confusion and disorientation over the past year. He has
diabetes mellitus and hypothyroidism, under good control. His blood work reveals a positive VDRL and FTA,
yet he has no history of syphilis. A CT scan of the brain is normal.

105. The next diagnostic test to obtain is


(A) MRI of the brain
(B) lumbar puncture
(C) cerebral angiograrn
(D) temporal artery biopsy
(E) electroencephalogram

106. If the test obtained is positive, management would include


(A) corticosteroids
(B) heparin
(C) acyclovir
(D) penicillin
(E) levothyroxine
Page 26 Level 3 Practice Items

CASE K

A 50-year-old male presents with fevers of 38.3-38.9"C (101-102°F) for the past month. He has been treated
with various antibiotics without response. On examination, he is alert but pale. Lungs reveal bilateral rales.
He has both nasal and oral ulcers. ECG reveals sinus tachycardia. Chest x-ray reveals bilateral nodules, some
of which are cavitary. Blood cultures are negative. Echocardiogram reveals normal cardiac valves. Complete
blood count reveals:

Hemoglobin: 10.3 g/dL (6.4 mmol/L)


Hematocrit: 0.31 (31%)
White blood cell count: 10,000/mm3 (10 x l o 9 k )
Segs: 81%
Lymphs: 10%
Monocytes: 9%

Sedimentation rate is 120 mmhr. ANA is positive, 1 :160 diffuse pattern.

107. The most likely diagnosis is


(A) systemic lupus erythematosus
(B) disseminated fungal infection
(C) Wegener's granulomatosis
@) metastatic carcinoma
(E) acute Staphylococcus aureus endocarditis

108. Which of the following will establish a definitive diagnosis?


(A) double-stranded DNA test
(B) lung biopsy
(C) transesophageal echocardiogram
(D) CT scan of the chest
(E) gallium scan

CASE L

A 53-year-old male presents with a history of bilateral scapular pain which episodically radiates into his upper
arms, more prominently on the left. He has noticed this discomfort for the past week after starting an aerobic
exercise program. The pain appears to be related to the intensity of the workouts (especially with lower body
workouts) resolving with rest, although he also complains of residual soreness. He also admits to "some short-
windedness" which he attributes to being out of shape. He has not seen a physician in the past ten years, and
past medical history is unremarkable. He was adopted and no family history is available. Physical
examination reveals a moderately obese male. Vital signs reveal a blood pressure of 160/90 mm Hg and a
pulse of 80/min, with the cardiac examination revealing an intermittent heart sound following S,. There is
mild muscular tenderness on range of motion of the upper extremities. The remainder of the examination is
unremarkable.
Level 3 Practice Items Page 27

109. The most appropriate action at this point would be


(A) reducing the intensity of his workout until he is better conditioned
(B) a trial of nonsteroidal anti-inflammatory drugs
(C) bilateral shoulder x-rays
(D) ECG and stress echocardiogram
(E) P-blocker therapy and blood pressure recheck in one month

110. Osteopathic manipulative treatment to effect the parasympathetic activity in this patient should be directed at
treatment of somatic dysfunction of the
(A) intercostais and upper ribs on the right
(B) intercostals and upper ribs on the left
(C) upper thoracics
(D) cervicals
(E) sacrum

CASE M

A 3-year-old male is brought to the office by his mother who noted a thick, yellow drainage from his left ear.
The child had a runny nose and dry cough for the previous week and complained of an earache for the Iast
two days. Examination of the tympanic membrane is obscured by the drainage in the ear canal. The child
denies any pain with movement of the pinna.

111. The most likely cause of the drainage is


(A) cerumen impaction
(B) otitis externa
(C) otitis media with tympanic membrane perforation
(D) chronic serous otitis media
(E) foreign body

112. The most appropriate management is


(A) irrigation of the ear canal in the office
(B) topical analgesic drops
(C) antibiotic otic suspension and oral antibiotics
(D) immediate surgical closure
(E) placement of ventilation tubes

113. The risk factor most likely to be related to the current problem is
(A) recurrent bouts of otitis media
(B) pre-term birth
(C) chronic sinusitis
(D) swimming in lake water
(E) the child's age
Page 28 Level 3 Practice Items

CASE N

A 40-year-old female presents to the office with a history of morning headaches and associated vomiting.

114. Physical examination reveals papilledema which is most likely secondary to


(A) ruptured intracranial aneurysm
(B) brain tumor
(C) partial seizure disorder
@) pregnancy
(E) trauma

115. The most useful tool in evaluating the patient would be


(A) cerebral angiogram
(B) MRI of the brain
(C) electroencephalogram
(D) ophthalmology consult
(E) lumbar puncture

CASE 0

An 18-year-old male presents to the office with a complaint of non-bloody diarrhea of seven days' duration.
He has a low-grade fever of 38.2"C (100.8"F) and lower abdominal cramps. The patient has no recent travel
history and he denies eating any shellfish or raw fish. The only abnormality on physical examination is that
the patient has significant facial acne, for which he has been using topical clindamycin 1% solution twice a
day for three to four months. The patient admits that his physician told him to use the solution only on an
intermittent basis, but not daily.

116. You suspect that the infectious organism causing the diarrhea is
(A) Clostridium dzficile
(B) Escherichia coli
(C) Giardia lamblia
(D) Salmonella typhi
(E) Shigella sonnei

117. The most appropriate treatment for this patient's diarrhea is


(A) intravenous antibiotics
(B) oral erythromycin
(C) oral rehydration
(D) oral metronidazole
(E) supportive care
Level 3 Practice Items Page 29

A 68-year-old male with a history of vertebrobasilar insufficiency presents to the office complaining of neck
pain.

118. In order to decrease the likelihood of recurrent symptoms, the patient should be advised against
(A) using a computer
(B) painting ceilings
(C) shoveling snow
(D) treadmill exercise
(E) planting a garden

119. Which of the following osteopathic manipulative treatments carries the greatest risk to the vertebral artery in
this patient?
(A) counterstrain treatment of the anterior first cervical tenderpoint
(B) muscle energy treatment of atlanto-axial dysfunction
(C) HVLA treatment of C7 extended, rotated right, sidebent right
(D) suboccipital tension release
(E) HVLA treatment of atlanto-axial dysfunction

CASE Q

A 21-year-old male presents to the emergency department via ambulance after being hit on the head with a
pipe during a street fight.

120. Which of the following is consistent with a Glasgow coma scale score of 13-15 in this patient?
(A) decerebrate posture
(B) localizes to painful stimuli
(C) decorticate posture
(D) grand ma1 seizure
(E) responds only with grunts and moaning

The patient was initially lucid with a non-focal neurologic examination, but began to complain of a severe
headache, dizziness, and nausea, and he began vomiting. He has since become unresponsive. A CT scan of
the brain would most likely show
(A) cerebral edema
(B) linear skull fracture
(C) epidural hematoma
(D) subdural hematoma
(E) basilar skull fracture
Page 30 Level 3 Practice Items

CASE R

A previously healthy 35-year-old sedentary male complains of the recent onset of palpitations. The proximate
cause for his visit was an episode of having collapsed while reaching overhead to change a light fixture.
Work-up in the emergency department yields a finding of occasional premature ventricular contractions with
a brief run of supraventricular tachycardia on electrocardiogram. The patient is given a prescription for
digoxin and is referred to a cardiologist. The patient notes that this problem of two weeks' duration follows
a rear-end motor vehicle accident wherein he suffered an acceleration/decelerationimpact while in a low-back
passenger seat with his hands clasped behind his head.

122. Given this history of mechanism of injury and subsequent symptoms, in w h c h of the following anatomical
regions would you expect to find acute tissue texture changes and motion restriction?
(A) C5-C7
(B) TI-T4
(C) T5-T8
(D) T10-L2
(E) L4-S1

123. This anatomical region is implicated in this patient's symptoms because it is the site of
(A) cervical whiplash fulcrum
(B) sacral whiplash fulcrum
(C) parasympathetics to the heart
(D) sympathetics to the heart
(E) transition from kyphosis to lordosis

CASE S

A 55-year-old male presents to the office with a chief complaint of severe constipation of six months'
duration. The patient claims that his stools have been dark maroon in color with a bad odor. The patient has
noted a 30-pound weight loss over the past six months even though his appetite has decreased only slightly.
Physical examination reveals a palpable right lower quadrant mass that is freely movable, but tender to
palpation. The rest of his physical examination, including evaluation of the rectum and genitalia, is normal.
The patient has no history of fever or chills.

124. The most likely diagnosis is


(A) Peutz-Jeghers syndrome
(B) colon carcinoma
(C) Crohn's disease
(D) ulcerative colitis
(E) appendiceal abscess
Level 3 Practice Items Page 31

125. The most appropriate surgical intervention would be


(A) jejunal resection
(B) total colectomy
(C) cecal resection
(D) right hemicolectomy
(E) appendectomy with abscess drainage

126. The most appropriate way to monitor for recurrence would be


(A) barium enema with air contrast
(B) hemoccult testing
(C) repeat CT scan of the abdomen
(D) diagnostic iaparoscopy
(E) surveillance colonoscopy

CASE T

A 57-year-old female presents with fatigue, a weight loss of ten pounds, and increasing mid-back pain over
the past six months. On examination, a pigmented velvety-appearing lesion is noted on the abdomen. The
paraspinal muscles are rigid bilaterally from the iliac crests to between the scapulae. Thoracic spine x-rays are ;

negative. Recent episodes of segmental areas of phlebitis of the lower extremities are reported.

127. Which of the following diagnostic studies would be the most appropriate initial work-up for this patient?
(A) Epstein-Barr titer
(B) ultrasound of the gallbladder
(C) flat plate x-ray of the abdomen
(D) CT scan of the abdomen
(E) rheumatoid factor

128. Further evaluation of the patient demonstrates abnormal AST, ALT, and LDH with splenomegaly. The most
likely diagnosis is
(A) common bile duct obstruction by a gallstone
(B) carcinoma of the body of the pancreas
(C) hepatitis C
(D) Addison's disease
(E) rheumatoid arthritis

129. Relief of back pain would best be approached with


(A) inhibitory pressure to paravertebral areas
(B) high velocity, low amplitude to the lumbodorsal area
(C) narcotic analgesics
(D) ultrasound to the paraspinal muscles
(E) nonsteroidal anti-inflammatory drugs
Page 32 Level 3 Practice Items

CASE U

A 23-year-old nervous-appearing female presents to the office complaining of paroxysmal palpitations, sweaty
palms, and dyspnea. She is afebrile, and vital signs reveal a blood pressure of 130182 mm Hg, a pulse of
112/min, and respirations of 20Imin.

130. On physical examination, which of the following would most likely have somatic dysfunction and tissue
texture changes?
(A) '4.4
(B) TI-T5
(C) TIO-TI1
(D) L2-L4
(E) sacrum

131. After a complete evaluation and a normal ECG, you decide that she has an anxiety disorder with autonomic
hyperstimulation. Which of the following is first-line therapy for this patient?
(A) P-blockers
(B) psychodynamic psychotherapy
(C) craniosacral therapy
(D) avoidance of caffeinated beverages
(E) buspirone

CASE V

A 34-year-old male presents to the office with interscapular pain. The pain started this morning after helping
his wife rearrange the furniture.

132. Which of the following findings is anticipated on physical examination?


(A) restriction in passive thoracic range of motion
(B) ropiness of the paravertebral muscles
(C) decrease in regional skin temperature
(D) local tissue atrophy
(E) parascapular muscle contracture

133. You diagnose T5 flexed, rotated left, sidebent left. In order to apply a direct technique to this somatic
dysfunction you would
(A) flex, rotate left, and sidebend left
(B) flex, rotate right, and sidebend right
(C) flex, rotate right, and sidebend left
(D) extend, rotate left, and sidebend right
(E) extend, rotate right, and sidebend right
Level 3 Practice Items Page 33

CASE W

One hour after dinner in his home, a 62-year-old male gradually slumped forward in his chair. After a few
moments he was arousable, but did not know his name and could not speak. He is brought to the emergency
department. He has a history of cirrhosis with ascites, and is status post coronary artery bypass grafting three
years ago. His medications include a P-blocker, furosemide, and aspirin. Physical examination is remarkable
only for his stigmata of cirrhosis and his mental status. He is oriented to person and can respond slowly to
verbal stimuli. He is lethargic and detached. He has no focal deficits. ECG and CT scan of his brain are
normal. His vital signs and serum analysis reveal:

Blood pressure:
Pulse:
Respirations:
Temperature:

Serum analysis:
Sodium:
Potassium:
Chloride:

Complete blood count:


White blood cell count: 11,300/mm3 (1 1.3 x 109L)
Hemoglobin: 15 g/dL (9.3 rnmol/L)
Segs: 72%
Lymphocytes: 25%

134. The next best diagnostic test to consider would be


(A) MRI of the brain
(B) serum bicarbonate level
(C) cerebral angiography
(D) serum ammonia level
(E) electroencephalogram

1 . The most appropriate approach to therapy would be


(A) intravenous hypertonic saline
(B) intravenous loading of phenytoin
(C) lactulose enema
(D) neurological consultation
(E) vascular surgery consultation

136. The pathophysiologic process underlying this patient's problem is


(A) global cerebral hypoxia
(B) toxic/metabolic disorder due to hepatocellular dysfunction and portosystemic shunting
(C) transient disturbance of cerebral function due to an abnormal paroxysmal neuronal discharge
(D) the age-related increase in antidiuretic hormone (ADH) responsiveness to stress
(E) microangiopathic degeneration of the cerebral small vasculature
Page 34 Level 3 Practice Items

CASE X

A 72-year-old male presents to the emergency department with a chief complaint of mid-epigastric abdominal
pain for ten days, fatigue, and melena, with one reported episode of maroon-colored stool at the onset of the
abdominal pain. Physical examination reveals mild epigastric tenderness and guaiac-positive soft brown stool.
He relates a history of seeing multiple physicians for his severe joint pain after retiring from his job at the
railroad. Laboratory studies performed in the emergency department reveal a hemoglobin of 9.0 g/dL (5.6
mmoLk), blood pressure of 95/50 mm Hg, and a heart rate of 1001min. Acute paraspinal changes are noted at
T5-T6.

137. Which of the following additional pieces of this patient's history would most likely be a positive contributing
factor to his current complaint?
(A) medications used
(El) family history of colon carcinoma
(C) allergies
(D) past surgical history
(E) employment history

138. The most appropriate modality for obtaining a diagnosis would be


(A) fiberoptic endoscopy
(B) upper gastrointestinal study with barium
(C) CT scan of the upper abdomen
(D) technetium-99m
(E) MRI of the abdomen

CASE Y

You are examining a 42-year-old secretary who is complaining of weakness in her right hand. This weakness
has been progressively developing over six months and is often accompanied by a sensation of numbness in
her index finger. She reports that she can no longer safely hold her coffee cup. On examination you identify
decreased sensation in her third finger. Her right hand grip is markedly diminished, Allen's test is negative,
Phalen's test is positive, and there is an inspiration somatic dyshnction of rib I on the right.

139. Which of the following tests is also likely to be positive?


(A) Finkelstein's test
(B) Spurling's test
(C) Tinel's test
(D) biceps reflex test
(E) hyperabduction test
Level 3 Practice Items Page 35

140. If you elect to treat rib 1 with high velocity low amplitude treatment, you should
(A) loosen the rib away from the barrier of motion by rotation of the head
(B) sidebend the torso away from the elevated rib
;c) position the nuchal line so that the muscles attached to rib 1 are lax
(D) have the patient take a deep inhalation and hold her breath prior to thrust
(E) apply cephalic to caudal thrust on the rib angle

141. Which of the following co-existing conditions would most likely be found in this patient?
(A) osteoarthritis
(B) scleroderma
(C) hypothyroidism
(D) diabetes
(E) muscular dystrophy

CASE Z

An 87-year-old retired professor presents to the office with his wife and daughter for evaluation of memory
loss. He denies any problems at all. His wife and daughter report increasing confusion jn the patient over the
.
past three years. The patient has been more irritable and moody and has stopped playing chess with his
friends. He has gotten lost several times driving to the grocery store two blocks away. On questioning, his
remote memory appears intact. although his judgment is poor. Mini Mental Status examination score is 15/30.
Past medical history includes mild hypertension. Physical examination is unremarkable.

142. The most likely diagnosis is


(A) dysthymia
(B) major depressive disorder
(C) delirium
(D) dementia of the Alzheimer's type
(E) occult malignancy

143. Your initial management strategies would include


(A) immediate hospitalization
(B) antipsychotic medication for confusion
(C) institutionaiization in an extended care facility
(D) increased daily structure
(E) a selective serotonin reuptake inhibitor

144. The current FDA-approved therapeutic option for the treatment of this patient includes
(A) antioxidants
(B) anti-inflammatory agents
(C) cholinesterase inhibitors
(D) estrogens
(E) ginkgc biloba
Page 36 Level 3 Practice Items

CASE AA

A 35-year-old female presents to the office complaining of left lower abdominal pain with abdominal bloating
and distention. She is experiencing diarrhea alternating with constipation. The diarrhea is the primary
complaint, and it occurs immediately after eating and also with stress. The patient denies weight loss,
vomiting or nocturnal awakening. Routine diagnostic studies, including complete blood count with
differential, erythrocyte sedimentation rate, blood chemistries, and stool examinations for white blood cells,
ova and parasites are all normal.

145. The next most appropriate diagnostic test is


(A) upper gastrointestinal study with barium swallow
(B) rectal biopsy
(C) ultrasound of the liver and gallbladder
@) colonoscopy
(E) fecal occult blood test

146. The diagnostic test performed is negative. You reassure the patient and discuss appropriate management,
which is
(A) fiber supplements
(B) oral omeprazole
(C) oral cisapride
(D) lactase
(E) oral H, blockers

An 8-year-old male with a known seizure disorder is being seen by paramedics. The child was actively
seizing on their arrivaI.

147. The child arrives in the emergency department unresponsive and without seizure activity. Vital signs reveal:

Pulse: 120imin; strong


Respirations: 20lmin; shallow
Blood pressure: 110 mm Hg systolic

Which of the following is the most appropriate initial approach after providing supplemental oxygen?
(A) intravenous phenytoin
(B) check finger stick glucose
(C) endotracheal intubation
(D) observe while awaiting anticonvulsant levels
(E) proceed to the waiting room to obtain a history from the mother
Level 3 Practice Items Page 37

148. The mother tells you thar the child had seized intermittently for 30 minutes before the paramedics arrived.
While obtaining this history, the child begins to seize again. You should
(A) administer lorazepam
(B) administer phenobarbital
(C) administer phenytoin
(D) consult a pediatrician
(E) reassure the mother and observe

149. The child remains unresponsive with occasional twitching of the extremities. The next step in management is
(A) diazepam
(B) observation
(C) lidocaine
(D) phenobarbital
(E) phenytoin

CASE CC

A 6-year-old male is hospitalized with bloody stools and a small bowel obstruction. Technetium scanning of
the abdomen reveals focal increased uptake in the ileum.

150. The most likely diagnosis is


(A) Hirschsprung's disease
(B) Crohn's disease
(C) Meckel's diverticulum
(D) mesenteric lymphadenitis
(E) celiac sprue

15 1. This condition is a result of


(A) a persistent omphalomesenteric duct
(B) excessive fat intake
(C) abnormal innervation of the abdominal wall
(D) calcium deficiency
(E) abnormal digestive enzymes

152. The patient undergoes surgical intervention and suffers a post-operative ileus. Bowel function will most likely
be enhanced by treating somatic dysfunction at which of the following levels?
(A) C4-C5
(B) C6-C7
(C) TI-T4
(D) T5-L2
(E) L3-L5
Page 38 Level 3 Practice Items

CASE DD

A 3 1-year-old male presents to the psychiatric inpatient unit complaining that the devil is telling him to go out
and kill people. He is able to name specific people when asked. His speech is disorganized and incoherent at
times. His mother reports that he is unable to care for himself.

153. The most appropriate medication for this patient is


(A) lorazepam
(B) buspirone
(C) tacrine
(D) haloperidol
(E) fluoxetine

154. The primary neurotransmitter involved in this patient's disorder is


(A) acetylcholine
(B) dopamine
(C) gamma-arninobutyric acid
@) norepinephrine
(E) serotonin

CASE EE

A 5-year-old male presents with headaches, hyperactive behavior, and learning difficulties six months after
falling and striking the back of his head. Neurologic examination is normal. Physical examination reveals a
parallelogram-shaped head, with his right eyeball and forehead appearing more prominent than the left. CT
scan and MRI of the brain are normal.

155. This patient's symptoms may result from which of the following cranial dysfunctions?
(A) superior vertical strain
(B) left torsion
(C) left lateral strain
(D) inferior vertical strain
(E) right lateral strain

156. This patient's cranial dysfunction will be most appropriately treated using
(A) high velocity low amplitude technique
(B) counterstrain
(C) muscle energy technique
@) indirect technique
(E) high velocity high amplitude technique
Level 3 Practice Items Page 39

CASE FF

A 72-year-old male is brought to the emergency department by his family. They state that he has a past
medical history of multiple myeloma and recently has been complaining of back pain, constipation, and
fatigue. His family states that he has "not been himself mentally."

157. Which of the following laboratory tests would be most crucial in the emergency evaluation of this patient?
(A) creatinine level
(B) serum globulin level
(C) complete blood count
(D) calcium level
(E) magnesium level

158. Which of the following therapies is most appropriate to institute in the emergency department?
(A) potassium replacement
(B) antibiotics
(C) calcium replacement
(D) blood transfusion
(E) saline infusion

CASE GG

A 72-year-old female with a history of early Alzheimer's disease presents to the office complaining of poor
sleep. She relates waking up at 2:00 a.m. thinking it was morning and being unable to fall back to sleep. She
also notes no interest in eating, as well as losing some interest in activities.

159. The most likely etiology of this patient's sleep disturbance is


(A) progressive dementia
(B) daylnight confusion
(C) anxiety
(D) depression
(E) psychosis

160. The most appropriate medication to help this patient sleep is


(A) trazodone
(B) diphenhydramine
(C) lorazepam
(D) haloperidol
(E) zolpidem
Page 40 Level 3 Practice Items

CASE HH

A 28-year-old metal worker presents to the office complaining of right eye pain, along with photophobia and
tearing for two days. He also states that he has a foreign body sensation in the same eye.

161. Which of the following would most likely be seen on slit lamp or Wood's lamp examination?
(A) dendritic lesions
(B) purulent discharge
(C) a "rust ring"
(D) a hazy cornea
(E) clouding of the anterior chamber

162. Appropriate management of this patient may include


(A) copious irrigation with 0.9% saline
(B) x-ray of the involved eye
(C) complete bed rest with head elevation
(D) admission with intravenous antibiotics
(E) tonometric pressure readings

CASE I1

A 6-year-old child is brought to the emergency department in a coma of undetermined etiology. Glasgow
coma scale score is 5, and the child manifests no cough or gag reflex. Respiratory rate is 4lmin and the lips
are cyanotic.

163. You note the child's blood alcohol level to be 480 mg/dL (1 15 mmol/L), and the parents admit that they had
left glasses and bottles from their previous night's party where the child had access to them. Appropriate
treatment at this point consists of
(A) dialysis
(B) endotracheal intubation
(C) gastric lavage
(D) N-acetylcysteine
(E) magnesium sulfate

164. The patient's mental status is unchanged by your therapeutic maneuver. Appropriate treatment now consists
of
(A) dialysis
(B) glucose administration
(C) gastric lavage
(D) N-acetylcysteine
(E) magnesium sulfate
Level 3 Practice Items Page 41

You are asked to evaluate a family who has recently arrived in this country. The family consists of a mother,
two grown sons, and a teenage daughter. Attempts to achieve a detailed history are hampered by language
differences. One of the sons has a cough, but is a non-smoker.

165. The most appropriate question to ask of the son is whether he has experienced
(A) change in vision
(B) recent skin rashes
(C) recent weight loss
(D) difficulty walking
(E) bowel or bladder problems

166. After completing a history and physical examination, which of the following skin tests should be performed on
each member of the family?
(A) scratch test
(B) Schick test
(C) Mantoux test
(D)tine test
(E) patch test

CASE KK

A 52-year-old female presents with right upper quadrant pain that has been present over the past few days,
which is getting worse with each day. On physical examination, you note increased tissue sensitivity along
TX-T9 on the right. You also elicit a positive Murphy's sign.

167. The most likely diagnosis is


(A) splenomegaly
(B) gastroesophageal reflux
(C) ischemic bowel
(D)acute cholecystitis
(E) irritable bowel syndrome

168. The most appropriate treatment for this patient is


(A) increased caloric intake
(B) an alcohol withdrawal program
(C) bowel resection
(D) cholecystectomy
(E) corticosteroids
Page 42 Level 3 Practice Items

CASE LL

A 30-year-old male prisoner presents with severe perianal pruritus. Physical examination reveals irregular
lesions at the anus.

169. The most likely diagnosis is


(A) AIDS
(B) gonococcal disease
(C) human papillomavirus
(D) non-specific proctitis
(E) cryptitis

170. The patient relates a history of drainage above the anus. A punctate opening is noted in the region of the
sacrum. The most likely diagnosis is
(A) inflammatory bowel disease with fistula
(El) cryptitis with sinus tract
(C) rectoperineal sinus tract
(D) pilonidal cyst
(E) ulcerated colitis with sinus tract

CASE MM

A 73-year-old female complains of mild to moderate left lower quadrant abdominal pain for the past two
weeks. She states that she has had some constipation in that time and some stools have been maroon-colored.

171. The diagnostic intervention that is the most specific in diagnosing this patient's chief complaint is
(A) hemoccult testing
(B) barium enema
(C) flexible colonoscopy
(D) MRI of the pelvis
(E) fiberoptic laparoscopic examination

172. Osteopathic structural findings would most likely include


(A) non-specific lower left lumbar restrictive motion changes
(B) left-on-left sacral torsion
(C) sacrococcygeal joint restriction
(D) T12-L1 somatic dysfunction on the left
(E) posteriorly rotsted left innominate
Level 3 Practice Items Page 43

173. Management of this patient, in addition to osteopathic manipulative treatment, should include
(A) antibiotics
(B) antidiarrheais
(C) antispasmodics
(D) counseling
(E) liquid dietary supplements

171. The patient develops a temperature of 38.1eC (100.5"F), rebound tenderness, and leukocytosis. The most
appropriate immediate treatment is
(A) surgical intervention via laparotomy incision
(B) enemas and digital disimpaction
(C) high-fiber diet and bulk-forming laxatives
(D) mesenteric lift techniques
(E) intravenous antibiotics and bowel rest

CASE NN

A 24-year-old female presents with alternating diarrhea and constipation exacerbated by stress but without
weight loss.

175. Anterior Chapman's points in this patient are expected to be palpated


(A) at the second intercostal space on the left
(B) at the periumbilical region
(C) along the iliotibial band
(D) on the superior pubic ramus
(E) in the fifth intercostal space bilaterally

176. Osteopathic manipulative treatment in the form of ganglion inhibition could impact this patient's left colon to
modulate sympathetic tone. The ganglion to be treated is
(A) ganglion in~par
(B) inferior mesenteric
(C) stellate
(D) celiac
(E) superior mesenteric

177. In this patient, the autonomic nervous system tone to the colon would best be described as
(A) decreased parasympathetics, increased sympathetics
(B) increased parasympathetics, decreased sympathetics
(C) increased parasympathetics, increased sympathetics
(D) decreased parasympathetics, decreased sympathetics
(E) unaltered
Page 44 Level 3 Practice Items

CASE 00

A 20-year-old college student presents with a complaint of cough and fever for the past three days. She states
that her coughing is non-productive and admits to a headache and myalgias. She relates a history of a "cold"
prior to the onset of her cough. The cough keeps her awake at night. Examination reveals a temperature of
37.7"C (99.8'F), reddened nasal mucosa with swollen inferior turbinates, and mild erythema of the pharynx.
Chest examination reveals clear lung sounds; acute tissue texture changes bilaterally at the level of T2-T4; T4
flexed, sidebent right, rotated right; and palpable, small, tender, anterior chain cervical lymph nodes.

178. The most likely diagnosis is


(A) bacterial bronchitis
(B) Haemophilus inJuenzae pneumonia
(C) Mycoplasmapneumoniae pneumonia
(D) pleurisy
(E) pneumococcal pneumonia

179. Management includes mobilization of the T2-T4 segments. The primary rationale for beginning treatment in
this area is that
(A) autonomic activity through the phrenic nerve will be normalized
(B) sympathetic hyperactivity from facilitated segments will be normalized
(C) sympathetic activity will be enhanced to permit thinning of secretions
(D) the diaphragm will have a rhythmic motion to improve ventilation
(E) the parasympathetic influence to the lungs will be normalized

180. Which of the following chest x-ray findings is consistent with this diagnosis?
(A) calcified hilar lymph nodes
(B) diffuse infiltrates
( C ) lobar infiltrates
(D) segmental infiltrates
(E) upper lobe infiltrates with cavitation

181. In addition to thoracic osteopathic manipulative treatment, you perform a series of lymphatic treatments.
Which of the following techniques should be performed first?
(A) Galbreath's technique
(B) infra and supra hyoid node technique
(C) mild fascia1 release
(D) tapotement (clopping) of the thoracic outlet
(E) vacuum thoracic pump
Level 3 Practice Items Page 45

182. Fees charged for the osteopathic manipulative treatment provided for this patient are based on the
(A) age of the patient
(B) comorbidities present
(C) complexity of the treatment
(D) number of areas treated
(E) time it takes for treatment

CASE PP

A previously healthy 26-year-old male presents with the chief complaint of "I'm tired all the time, I have no
energy. I'm losing weight, and 1 think I might have cancer." He describes the onset as occurring four months
ago with no environmental precipitating factors. He denies problems with his social life. He describes that at
work he is getting behind and has to force himself to do everything. He admits to a decrease in the frequency
of intercourse, and states that he is becoming increasingly indecisive about even simple choices. He admits to
global sleep impairment "because I worry about everything." He also has a diminished appetite, which he
relates to some cramping abdominal pain and indigestion. He states that he feels depressed, and he exhibits
flexed posture, sighing respirations, and mild psychomotor retardation.

183. To which of the general DSM-IV diagnostic categories does this clinical presentation conform?
(A) thought disorder
(B) mood disorder
(C) anxiety disorder
(D) somatoform disorder
(E) hypochondriasis

184. Which of the following medications represents the best choice for an initial clinical trial?
(A) perphenazine-amitriptyline
(B) phenelzine
(C) trazodone
(D) paroxetine
(E) buspirone

185. Which of the following conditions could arise from the medical therapy of this patient's disorder?
(A) neuroleptic malignant syndrome
(B) sexual dysfunction
(C) adenomatous mitral valve prolapse
(D) nephrogenic diabetes insipidus
(E) tardive dyskinesia
Page 46 Level 3 Practice Items

CASE QQ

A 32-year-old female presents with the complaint of vaginal discharge, vaginal pruritus, and excoriation of the
vulva. On examination, a thick white vaginal discharge with erythema of the vulva and vagina is confirmed.

186. Which of the following procedures would most likely confirm the diagnosis?
(A) DNA probe for chlamydia
(B) DNA probe for gonorrhea
(C) Pap smear
(D) vaginal culture for aerobic and anaerobic organisms
(E) wet prep with saline and KOH solution

187. You expect the vaginal pH to be


(A) < 3.0
(B) 4.0 - 4.5
-
(C) 5.0 6.0
0)6.0 - 7.0
(E) > 7.0

CASE RR

A 50-year-old male presents to the emergency department from his home because of ataxia. His family
brought him in and they state that his only medical problem is that he has been a heavy abuser of alcohol for
many years. The patient is ataxic, confused, pale, malnourished, and has a very heavy smell of alcohol on his
breath.

188. Anemia found in this patient is due to a deficiency of


(A) iron
(B) folate
(C) erythropoietin
(D) thiamine
(E) magnesium

189. The primary pathway of ethanol oxidation in this patient's liver is via which of the following enzyme
systems?
(A) citric acid (TCA) cycle
(B) microsomal ethanol-oxidizing system
(C) catalase
(D) alcohol dehydrogenase
(E) cytochrome P,,, system
Level 3 Practice Items Page 47

CASE SS

A 25-year-old male became depressed after an accident in the lab last week in which a fellow graduate student
was burned. He felt that the accident was the lab supervisor's fault. He began to neglect his own research
project and became irritated and suspicious when reprimanded by his graduate advisor. He began to feel that
fellow students were talking about him and "sabotaging" his work. After a near-violent confrontation with the
lab supervisor, he was taken to the university emergency department and admitted. He seemed confused in his
thinking. He admitted to hearing voices intermittently saying "you're next." His affect was labile and his
cognitive functions were intact except for decreased concentration. He responded rapidly to supportive care
and low-dose haloperidol, which was discontinued at discharge.

190. The most likeiy diagnosis is


(A) brief psychotic disorder
(3)schizophreniform disorder
(C) paranoid schizophrenia
(D) delusional disorder
(E) post-traumatic stress disorder

191. Four months following his hospitalization, the patient is brought to the clinic by his parents, who describe him
as very withdrawn. He found part-time lab-related work but feels increasingly afraid. H e reports that
television news programs are trying to warn him of an impending accident but he "can't decipher the code."
His symptoms remit with restarting haloperidol but he requires a higher dose. His diagnosis at this point is
(A) brief psychotic disorder
(B) schizophreniform disorder
(C) paranoid schizophrenia
(D) delusional disorder
(E) post-traumatic stress disorder

192. Shortly after the increased haloperidol dosage, he becomes increasingly agitated and unable to sit still. He
paces constantly and reports "I just can't get comfortable." The most likely explanation for these symptoms is
(A) the precipitation of a manic attack
(B) neuroleptic malignant syndrome
(C) aggravation of an underlying anxiety disorder
@) akathisia
(E) neuroleptic-induced panic attack
Page 48 Level 3 Practice Items

CASE TT

A 25-year-old female presents to the hospital with severe vaginal bleeding. She is at 34 weeks' gestation and
is diagnosed as having an abruptio placentae. She begins to bleed from her intravenous sites as well as from
her nose and urethra. Blood work is done and results are listed below:

Hemoglobin: 9.7 g/dL (6.02 mmol/L)


Hematocrit: 0.282 (28.2%)
White blood cell count: 8,500/mm3 (8.5 x 109/L)
Platelet count: 24,000/mm3 (24 x 109/L)
Prothrombin time: 24 sec
INR: 2.5
Fibrinogen: 100 mg/dL ( I .O g/L)
Fibrin split products: > 40 mg/dL (0.40 g L )
Partial thromboplastin time: 90 sec

A peripheral smear shows red blood cell fragments.

193. The most likely diagnosis is


(A) Rh incompatibility
(B) idiopathic thrombocytopenic purpura
(C) thrombotic thrombocytopenic purpura
(D) disseminated intravascular coagulation
(E) hemophilia A

194. The most likely cause of this patient's bleeding disorder is


(A) underlying malignancy
(B) idiopathic
(C) abruptio placentae
(D) congenital deficiency of factor VIII
(E) congenital deficiency of von Willebrand's factor

195. The definitive therapy for this patient would be


(A) plasmapheresis
(B) delivery of the fetus
(C) platelet transfusions
(D) fresh frozen plasma
(E) high-dose corticosteroids

A 66-year-old male presents to the emergency department with a 5-cm laceration to his lower leg caused by a
shovel while working in his yard. The wound appears clean. Neurovascular examination is within normal
limits; however, subcutaneous tissue is exposed.
Level 3 Practice Items Page 49

196. This patient's wound care should include


(A) betadine scrub 10% mechanical scrubbing to the total wound
(B) copious irrigation with 0.9% sodium chloride
(C) hyperbaric wound therapy
(D) irrigation with 70% hydrogen peroxide
(E) local infiltration with a first-generation cephalosporin

197. This patient has no known drug allergies. Which of the following oral antibiotics is most appropriate?
(A) cephalexin
(B) metronidazole
(C) trimethoprim-sulfamethoxazole
(D) penicillin V potassium
(E) tetracycline

CASE W

A 17-year-old soccer player presents following a collision injury earlier in the day. He is on crutches and
complains of right thigh pain. He has no significant medical history and is otherwise feeling well. Physical
examination reveals a tender, non-fluctuant "mass-like" swelling in the anterior mid-thigh. Both passive and
active range of motion elicit pain. Muscle testing demonstrates weakness and painful contractions. The
patient can perform extension of the knee to approximately 30" and flexion to about 90'. Neurovascular
examination is within normal limits. X-rays of the thigh are negative for fracture.

198. The most likely diagnosis is


(A) complete quadriceps rupture
(B) rupture of the vastus intermedius and rectus femoris
(C) rupture of the sartorius
(D) quadriceps contusion with hematoma
(E) traumatic inflammation of a lipoma

199. The most appropriate course of action is


(A) immediate surgical repair
(B) immediate MRI
(C) ice, an elastic bandage, knee immobilizer and crutches
(D) aspiration of the mass
(E) heat and stretching along with a nonsteroidal anti-inflammatory drug

200. Myositis ossificans can best be prevented in this patient by


(A) early recognition of quadriceps rupture with immediate surgical repair
(B) non-weight-bearing status via crutches for a minimum of two weeks
(C) corticosteroid injections at the site of injury
(D) osteopathic manipulative treatment to the sacrum and pelvis
(E) an aggressive program of physical therapy
ANSWER KEY
NORMAL ADULT LABORATORY VALUES

BLOOD, PLASMA. SERUM


Alanine arninotransferase (ALT, SGPT at 37°C) . . . . . . . . . . . . . . . . . . . 5-35 mU/mL (5-35 UIL)
Albumin, serum . . . . . . . . . . . . . . . . . . . . . . . . . . . . . . . . . . . . . . . . .3.5-5.0 g/dL (35-50 g/L)
Ammonia, serum . . . . . . . . . . . . . . . . . . . . . . . . . . . . . . . . . . . . . . . . 15-45 pg/dL (1 1-32 pmol/L)
Amylase, serum . . . . . . . . . . . . . . . . . . . . . . . . . . . . . . . . . . . . . . . . . 25-125 mIU!mL (25-125 U!L)
Aspartate aminotransferase (AST, SGOT at 37'C) . . . . . . . . . . . . . . . . . . 7-40 rnU/mL (7-40 U 5 )
Bilirubin, serum
Adult: Total . . . . . . . . . . . . . . . . . . . . . . . . . . . . . . . . . . . . .0.2-1.3 mg/dL (3-22 pmol/L)
Direct . . . . . . . . . . . . . . . . . . . . . . . . . . . . . . . . . . . . . . . . .0.0-0.3 mddL (0-5 pmol/L)
Calcium, serum . . . . . . . . . . . . . . . . . . . . . . . . . . . . . . . . . . . . . . . . 9.0- 1 1 .Omg/dL (2.25-2.75 mmol1L)
Cholesterol, serum . . . . . . . . . . . . . . . . . . . . . . . . . . . . . . . . . . . . . . . . 140-250 mg/dL (3.6-6.5 mmolll)
HDL . . . . . . . . . . . . . . . . . . . . . . . . . . . . . . . . . . . . . . . . . > 35 mg/dL (> 0.9 mmol/L)
LDL . . . . . . . . . . . . . . . . . . . . . . . . . . . . . . . . . . . . . . . . .< 130 rng/dL (< 3.37 mn~ol/L)
Triglycerides, serum . . . . . . . . . . . . . . . . . . . . . . . . . . . . . . . . .40-1 50 mg/dL (0.4-1.5 g / L )
CO: content. blood . . . . . . . . . . . . . . . . . . . . . . . . . . . . . . . . . . . . . . 22-29 . mmol/L
Cortisol, plasma
8 AM . . . . . . . . . . . . . . . . . . . . . . . . . . . . . . . . . . . . . . . . . 6-23 pg/dL (170-635 nmol/L)
4 PM . . . . . . . . . . . . . . . . . . . . . . . . . . . . . . . . . . . . . . . . . 3-15 pgIdL (82-413 nmol1L)
10 PM . . . . . . . . . . . . . . . . . . . . . . . . . . . . . . . . . . . . . . . . .< 5056 of 8 AM value
Creatinine, serum . . . . . . . . . . . . . . . . . . . . . . . . . . . . . . . . . . . . . . . . 0.7-1.5 mg/dL (62-133 pmol/L)
Creat~ninephosphokinase . . . . . . . . . . . . . . . . . . . . . . . . . . . . . . . . . . .50-200 U&
Electrolytes, serum
Sodium . . . . . . . . . . . . . . . . . . . . . . . . . . . . . . . . . . . . . . . . . 136-146 mEq/L (136-146 mmolIL)
Chloride . . . . . . . . . . . . . . . . . . . . . . . . . . . . . . . . . . . . . . . . . 96- 106 mEq/L (96-1 06 mmol/L)
Potassium . . . . . . . . . . . . . . . . . . . . . . . . . . . . . . . . . . . . . . . . 3 5 4 . 5 mEq/L (3 5 4 . 5 mmol/L)
Bicarbonate . . . . . . . . . . . . . . . . . . . . . . . . . . . . . . . . . . . . . . . 23-29 mEq/L (23-29 mmol/L)
Ferritin, serum
Male . . . . . . . . . . . . . . . . . . . . . . . . . . . . . . . . . . . . . . . . . 27-270 nglmL (27-270 pglL)
Female . . . . . . . . . . . . . . . . . . . . . . . . . . . . . . . . . . . . . . . . . 9- 180 nglmL (9-180 pg/L)
Fibrinogen, plasma . . . . . . . . . . . . . . . . . . . . . . . . . . . . . . . . . . . . . . .200-400 mg/dL (5.9- 1 1.7 pmol/L)
Follicle-stimulating hormone, plasma
Male . . . . . . . . . . . . . . . . . . . . . . . . . . . . . . . . . . . . . . . . . 4-25 mU/mL (4-25 U L )
Female: Premenopause . . . . . . . . . . . . . . . . . . . . . . . . . . . . . . . 4-30 mU/mL (4-30 UIL)
Midcycle peak . . . . . . . . . . . . . . . . . . . . . . . . . . . . . . . 10-90 mU/mL (10-90 UIL)
Postmenopause . . . . . . . . . . . . . . . . . . . . . . . . . . . . . .40-250 mU/mL (40-250 UIL)
Gases, arterial blood (room air)
pH . . . . . . . . . . . . . . . . . . . . . . . . . . . . . . . . . . . . . . . . . 7.35-7.45
POz . . . . . . . . . . . . . . . . . . . . . . . . . . . . . . . . . . . . . . . . . 75-100 mm Hg (10.0-13.3 kPa)
PCO, . . . . . . . . . . . . . . . . . . . . . . . . . . . . . . . . . . . . . . . . .35-45 mm Hg (4.7-6.0 kPa)
0, saturation . . . . . . . . . . . . . . . . . . . . . . . . . . . . . . . . . . . . . .94- 100% (0.94- 1.00)
Glucose, serum
Fasting . . . . . . . . . . . . . . . . . . . . . . . . . . . . . . . . . . . . . . . . . 70-1 15 mg/dL (3.89-6.38 mmolk)
2 h postprandial . . . . . . . . . . . . . . . . . . . . . . . . . . . . . . . . . . . . < 120 mgIdL (< 6.7 mmolk)
Growth hormone . . . . . . . . . . . . . . . . . . . . . . . . . . . . . . . . . . . . . . . . . 0-10 ng!mL (0-10 pg/L)
f3-Human chorionic gonadotropin (male or non-pregnant female) . . . . . . . . < 3.0 IUIL
Immunoglobulins, serum
IgA . . . . . . . . . . . . . . . . . . . . . . . . . . . . . . . . . . . . . . . . . 60-333 mg/dL (0.6-3.3 g/L)
IgD . . . . . . . . . . . . . . . . . . . . . . . . . . . . . . . . . . . . . . . . . 0.5-3.0 mg/dL (5-30 m g L )
IgE . . . . . . . . . . . . . . . . . . . . . . . . . . . . . . . . . . . . . . . . . < 500 nglmL (< 500 pg/L)
IgG . . . . . . . . . . . . . . . . . . . . . . . . . . . . . . . . . . . . . . . . 550-1900
. mg/dL (5.5-19.0 g/L)
IgM . . . . . . . . . . . . . . . . . . . . . . . . . . . . . . . . . . . . . . . . .45-145 mg/dL (0.45-1.5 g/L)
Iron. serum . . . . . . . . . . . . . . . . . . . . . . . . . . . . . . . . . . . . . . . . . 75-175 pg/dL (13-31 ,umol/L)
Lactate dehydrogenase (LDH) . . . . . . . . . . . . . . . . . . . . . . . . . . . . . . . . 45-90 U/L
Luteinizing hormone, serumlplasma
Male . . . . . . . . . . . . . . . . . . . . . . . . . . . . . . . . . . . . . . . . . 6-1 8 mU/mL (6-1 8 U/L)
Female: Follicular phase . . . . . . . . . . . . . . . . . . . . . . . . . . . . . .5-22 mU/mL (5-22 U/L)
Midcycle . . . . . . . . . . . . . . . . . . . . . . . . . . . . . . . . . .three times baseline
Postmenopause . . . . . . . . . . . . . . . . . . . . . . . . . . . . . .> 30 mU1mL (> 30 UIL)
Magnesium, serum . . . . . . . . . . . . . . . . . . . . . . . . . . . . . . . . . . . . . . . 1.3-2.1 mEqiL (0.65-1.05 mmol/L)
Osmolality, serum . . . . . . . . . . . . . . . . . . . . . . . . . . . . . . . . . . . . . . . . 285-295 mOsm/kg serum water
Parathyroid hormone, serum, N-terminal . . . . . . . . . . . . . . . . . . . . . . . . . 230-630 pgimL (230-630 ngIL)
Phosphatase (alkaline), serum (p-NPP at 30'C) . . . . . . . . . . . . . . . . . . . . 20-90 mU/mL (20-90 U/L)
Phosphorus (inorganic), serum . . . . . . . . . . . . . . . . . . . . . . . . . . . . . . . . 3 .O-4.5 mgidL (1 .O- 1.5 mmoUL)
Prolactin, serum (hPRL) . . . . . . . . . . . . . . . . . . . . . . . . . . . . . . . . . . . . < 25 nglmL (< 25 pglL)
Proteins, serum
Total (recumbent) . . . . . . . . . . . . . . . . . . . . . . . . . . . . . . . . . . 6.0-8.0 gldL (60-80 g/L)
Albumin . . . . . . . . . . . . . . . . . . . . . . . . . . . . . . . . . . . . . . . . . 3.5-5.0 g/dL (35-50 g/L)
Globulin . . . . . . . . . . . . . . . . . . . . . . . . . . . . . . . . . . . . . . . . .0.2-1.7 g!dL (2-17 g 5 )
Thyroid-stimulating hormone (TSH) . . . . . . . . . . . . . . . . . . . . . . . . . . . 0.35-7.0 pU/mL (0.3 5-7.0 mUL)
Thyroxine (T,), serum . . . . . . . . . . . . . . . . . . . . . . . . . . . . . . . . . . . . . 5-12 pg/dL (65-155 nmoliL)
Triiodothyronine (T,), serum (RIA) . . . . . . . . . . . . . . . . . . . . . . . . . . . . 150-250 ng/dL (2.3-3.9 nmol/L)
Triiodothyronine (T,), resin uptake . . . . . . . . . . . . . . . . . . . . . . . . . . . .25-38% uptake (0.25-0.38 uptake)
Troponin-I . . . . . . . . . . . . . . . . . . . . . . . . . . . . . . . . . . . . . . . . .< 0.35nglmL
Urea nitrogen, serum (BUN) . . . . . . . . . . . . . . . . . . . . . . . . . . . . . . . . . 1 1-23 mg1dL (7.9- 16.4 mmol1L)
Uric acid, serum . . . . . . . . . . . . . . . . . . . . . . . . . . . . . . . . . . . . . . . . .2.6-7.2 mg/dL (0.15-0.42 mmol/L)

CEREBROSPINAL FLUID
Cell count . . . . . . . . . . . . . . . . . . . . . . . . . . . . . . . . . . . . . . . . . < 5imm3
Chloride . . . . . . . . . . . . . . . . . . . . . . . . . . . . . . . . . . . . . . . . . 120-130 mEq/L (120-130 mmolL)
Gamma globulin . . . . . . . . . . . . . . . . . . . . . . . . . . . . . . . . . . . . . . . . . 3- 12% (0.03-0.12)
Glucose . . . . . . . . . . . . . . . . . . . . . . . . . . . . . . . . . . . . . . . . . 50-75 mgldL (2.8-4.2 mmol1L)
Pressure . . . . . . . . . . . . . . . . . . . . . . . . . . . . . . . . . . . . . . . . .70-180 mm H,O
Protein . . . . . . . . . . . . . . . . . . . . . . . . . . . . . . . . . . . . . . . . .15-45 mg1dL (0.15-0.45 g/L)

HEMATOLOGIC
. Bleeding time (template) . . . . . . . . . . . . . . . . . . . . . . . . . . . . . . . . . . . 2.5-9.5 min
Erythrocyte count, blood
Male . . . . . . . . . . . . . . . . . . . . . . . . . . . . . . . . . . . . . . . . . 4.6-6.2 milIion/n~~n~
(4.2-6.2 x 10'2L)
Female . . . . . . . . . . . . . . . . . . . . . . . . . . . . . . . . . . . . . . . . . 4.2-5.4 million/mm3 (4.2-5.4 x lo"&,)
Hematocrit
Male . . . . . . . . . . . . . . . . . . . . . . . . . . . . . . . . . . . . . . . . . 40-54% (0.40-0.54)
Female . . . . . . . . . . . . . . . . . . . . . . . . . . . . . . . . . . . . . . . . 37-47%. (0.37-0.47)
Hemoglobin
Male . . . . . . . . . . . . . . . . . . . . . . . . . . . . . . . . . . . . . . . . . 14.0-18.0 g/dL (2.17-2.79 mmol/L)
Fema!e . . . . . . . . . . . . . . . . . . . . . . . . . . . . . . . . . . . . . . . . . 12.0- 16.0 g/dL (1.86-2.48 mmolIL)
Hemoglobin, glycosylated . . . . . . . . . . . . . . . . . . . . . . . . . . . . . . . . . . . 5.3-7.5%
Hemoglobin, plasma . . . . . . . . . . . . . . . . . . . . . . . . . . . . . . . . . . . . . .0-5 mg/dL (0-0.8 pmol/L)
Leukocyte count and differential
Leukocyte count . . . . . . . . . . . . . . . . . . . . . . . . . . . . . . . . . . . 4500-1 1,000/mm3(4.5-1 1.0 x lO7/L)
Segmented neutrophils . . . . . . . . . . . . . . . . . . . . . . . . . . . . . . . 54-62% (0.54-0.62)
Bands . . . . . . . . . . . . . . . . . . . . . . . . . . . . . . . . . . . . . . . . 3-5%
. (0.03-0.05)
Eosinophils . . . . . . . . . . . . . . . . . . . . . . . . . . . . . . . . . . . . . . . 1-3% (0.0 1-0.03)
Basophils . . . . . . . . . . . . . . . . . . . . . . . . . . . . . . . . . . . . . . . . 0-0.75% (0-0.0075)
Lymphocytes . . . . . . . . . . . . . . . . . . . . . . . . . . . . . . . . . . . . . . 25-33% (0.25-0.33)
Monocytes . . . . . . . . . . . . . . . . . . . . . . . . . . . . . . . . . . . . . . . 3-7% (0.03-0.07)
Mean Corpuscular Hemoglobin . . . . . . . . . . . . . . . . . . . . . . . . . . . . . . . 27-3 1 pg
Mean Corpuscular Hemoglobin Concentration . . . . . . . . . . . . . . . . . . . . . 33-37 g/dL (330-370 giL)
Mean Corpuscular Voiume . . . . . . . . . . . . . . . . . . . . . . . . . . . . . . . . .80-96 /*' (80-96 fL)
Partial thromboplastin r i i i e (activated) . . . . . . . . . . . . . . . . . . . . . . . . . . 20-35 sec
Platelet count, blood . . . . . . . . . . . . . . . . . . . . . . . . . . . . . . . . . . . . . .150,000-350,000/mm3(150-350 x 109iZ)
Prothrombin time . . . . . . . . . . . . . . . . . . . . . . . . . . . . . . . . . . . . . . . 12.0-14.0 sec
Red blood cell count . . . . . . . . . . . . . . . . . . . . . . . . . . . . . . . . . . . . . . 4.2-6.2 million/mm3 (4.2-6.2 x 10'2/L)
Red blood cell distribution . . . . . . . . . . . . . . . . . . . . . . . . . . . . . . . . . . - < 14.5%
Red cell (volume)
Male . . . . . . . . . . . . . . . . . . . . . . . . . . . . . . . . . . . . . . . . .20-36 m L k g (0.020-0.036 Lkg)
Female . . . . . . . . . . . . . . . . . . . . . . . . . . . . . . . . . . . . . . . . 19-3
. 1 m L k g (0.019-0.03 1 L k g )
Reticulocyte count . . . . . . . . . . . . . . . . . . . . . . . . . . . . . . . . . . . . . . . . 0.5-1.5% of erythrocytes
Sedimentation rate, erythrocyte (Westergren)
Male . . . . . . . . . . . . . . . . . . . . . . . . . . . . . . . . . . . . . . . . . 0-1 5 mm in one hour (0-15 mmih)
Female . . . . . . . . . . . . . . . . . . . . . . . . . . . . . . . . . . . . . . . . .0-20 mm in one hour (0-20 mmlh)
Thrombin time . . . . . . . . . . . . . . . . . . . . . . . . . . . . . . . . . . . . . . . . . < 2 seconds deviation from control
Volume, plasma
Male . . . . . . . . . . . . . . . . . . . . . . . . . . . . . . . . . . . . . . . . 25-43
. mLkg (0.025-0.043 Llkg)
Female . . . . . . . . . . . . . . . . . . . . . . . . . . . . . . . . . . . . . . . . . 28-45 mL/kg (0.028-0.045 Lkg)
White blood cell count . . . . . . . . . . . . . . . . . . . . . . . . . . . . . . . . . . . . . 3200-9800/mm3 (3.2-9.8 x 109/L)

SEMEN
Volume . . . . . . . . . . . . . . . . . . . . . . . . . . . . . . . . . . . . . . . . .2-6mL
Motility (grades 0-4) . . . . . . . . . . . . . . . . . . . . . . . . . . . . . . . . . . . . . .>50%
Count (millions1mL) . . . . . . . . . . . . . . . . . . . . . . . . . . . . . . . . . . . . . . 20- 100

SWEAT
Chloride . . . . . . . . . . . . . . . . . . . . . . . . . . . . . . . . . . . . . . . . . 0-30 mmol/L

URINE
Calcium . . . . . . . . . . . . . . . . . . . . . . . . . . . . . . . . . . . . . . . . . < 250 mgl24 h (< 6.3 nmoil24 h)
Chloride . . . . . . . . . . . . . . . . . . . . . . . . . . . . . . . . . . . . . . . . . 1 10-250mmolld (varies with intake)
Creatinine clearance
Male . . . . . . . . . . . . . . . . . . . . . . . . . . . . . . . . . . . . . . . . . 1 10-150 mL/min
Female . . . . . . . . . . . . . . . . . . . . . . . . . . . . . . . . . . . . . . . . . 105-132 mL/min
Estriol, total (in pregnancy)
30 wks . . . . . . . . . . . . . . . . . . . . . . . . . . . . . . . . . . . . . . . . . 6-18 mg!24 h (21-62 pm01124 h)
35 wks . . . . . . . . . . . . . . . . . . . . . . . . . . . . . . . . . . . . . . . . . 9-28 mg/24 h (31-97 pm01124 h)
40 wks . . . . . . . . . . . . . . . . . . . . . . . . . . . . . . . . . . . . . . . . . 13-42 mg/24 h (45-146 ,urn01124 h)
17-Hydroxycorticosteroids
Male . . . . . . . . . . . . . . . . . . . . . . . . . . . . . . . . . . . . . . . . . 3-9 mg/24 h (8.3-25 /*moll24 h)
Female . . . . . . . . . . . . . . . . . . . . . . . . . . . . . . . . . . . . . . . . . 2-8 rngl24 h (5.5-22 ,~moil24h)
Ketosteroids
Male . . . . . . . . . . . . . . . . . . . . . . . . . . . . . . . . . . . . . . . . . 6-1 8 mg/24 h (2 1-62 pmo1124 h)
Female . . . . . . . . . . . . . . . . . . . . . . . . . . . . . . . . . . . . . . . . .4-13 mgl24 h (14-45 pmo1124 h)
Osmolality . . . . . . . . . . . . . . . . . . . . . . . . . . . . . . . . . . . . . . . . . 38-1400 mOsm/kg water
PH . . . . . . . . . . . . . . . . . . . . . . . . . . . . . . . . . . . . . . . . . 4.6-8.0
Potassium . . . . . . . . . . . . . . . . . . . . . . . . . . . . . . . . . . . . . . . . . 25-125 mEq/d (varies with diet)
Proteins, total . . . . . . . . . . . . . . . . . . . . . . . . . . . . . . . . . . . . . . . . . 10-150 mgl24 h
Sodium . . . . . . . . . . . . . . . . . . . . . . . . . . . . . . . . . . . . . . . . . 40-220 mEq1d (varies with diet)
Specific gravity . . . . . . . . . . . . . . . . . . . . . . . . . . . . . . . . . . . . . . . . . 1.003- 1.030
Uric acid . . . . . . . . . . . . . . . . . . . . . . . . . . . . . . . . . . . . . . . . . varies with diet

Anda mungkin juga menyukai